PM2020 I+Endocrinology
PM2020 I+Endocrinology
PM2020 I+Endocrinology
lncretin inhibitor
Alpha-glucosidase inhibitor
Glucagon inhibitor
Submit answer
Reference ranges v
D1pept 1dyl pept idase-4 (DPP-4) 1nht61tor
Alpha-glucosidase inhibitor
Glucagon inhibitor
Next question )
Liraglutide is the other GLP-1 mimetic currently available. One the main
advantages of liraglutide over exenatide is that it only needs to be given
once a day.
Key points
• oral preparation
• trials to date show that the drugs are relatively well tolerated with
no increased incidence of hypoglycaemia
• do not cause weight gain
Save my notes
Q2
A 33-year-old man with tingling in his t humb, index and middle finger
has been referred for endocrine consultation. He also complains of
waking up incredibly tired and says his wife comp lains that he snores.
On examination, he is noted to have a prominent brow ridge. Looking at
old photos, it becomes clear that his facia l appearance has drastically
changed over time. After some blood tests and an MRI scan, he is
prescribed octreotide. What is the mechanism of action of this drug?
Somatostatin analogue
Dopamine agonist
Dopamine antagonist
Submit answer
Reference ranges v
prescribed octreotide. What is the mechanism of action of this drug?
I Somatostatln analogue
Dopamine agonist
Dopamine antagonist
••
11
f • Discuss (1) Improve
Next question )
Next question >
Acromegaly: management
Dopamine agonists
• for example bromocriptine
• the first effective medical treatment for acromegaly, however now
superseded by somatostatin analogues
• effective only in a minority of patients
Somatostatin analogue
• directly inhibits the release of growth hormone
• for example octreotide
• effective in 50-70% of patients
• may be used as an adjunct to surgery
Pegvisomant
• GH receptor antagonist - prevents dimerization of the GH receptor
• once daily s/c administration
• very effective - decreases IGF-1 levels in 90% of patients to normal
• doesn't reduce tumour volume therefore surgery still needed if
mass effect
Next question )
Q3
Nifedipine
Atorvastatin
Aspirin
Senna
Ferrous sulphate
Submit answer
Reference ranges v
Atorvastatin
Aspirin
Senna
Ferrous sulphate
Key points
• initial starting dose of levothyroxine should be lower in elderly
patients and those with ischaemic heart disease. The BNF
recommends that for patients with cardiac disease, severe
hypothyroidism or patients over 50 years the initial starting dose
should be 25mcg od with dose slowly titrated. Other patients
should be started on a dose of 50-1 00mcg od
• following a change in thyroxine dose thyroid function tests should
be checked after 8-12 weeks
• the therapeutic goal is 'normalisation' of the thyroid stimulating
hormone (TSH) level. As the majority of unaffected people have a
TSH value 0.5-2.5 mU/I it is now thought preferable to aim for a
TSH in this range
• women with established hypothyroidism who become pregnant
should have their dose increased 'by at least 25-50 micrograms
levothyroxine'* due to the increased demands of pregnancy. The
TSH should be monitored carefully, aiming for a low-normal value
• there is no evidence to support combination therapy with
levothyroxine and liothyronine
Interactions
• iron, calcium carbonate
o absorption of levothyroxine reduced, give at least 4 hours
apart
Q4
0 5.7
2 7.6
Normal
Diabetes mellltus
Subm it answer
Reference ranges v
Normal
Diabetes mellitus
Both the fasting and two-hour glucose are within normal limits.
Next question )
CamSc.an,ner ~ '....,.i;lo-" ~ ~ I
Fasting glucose: a Ommoll >• 7,0 mmol,I
Horwalwu-iccontnll
M:IA It rs ~:'..'7 ITTnohnel II 0-e 4 l
Cl
~ I • "'G 'II 6 Hl 9 IM'IO'.I
A fasting glucose greater than or equal to 6.1 but less than 7.0 mmol/I
implies impaired fasting glucose (IFG)
Diabetes UK suggests:
• 'People with IFG should then be offered an ora l glucose tolerance
test to rule out a diagnosis of diabetes. A resu lt below 11 .1
mmol/I but above 7.8 mmol/1 indicates that the person doesn't
have diabetes but does have IGT.'
Next question )
B I ~ A ..... ·-·- 1-
i: =
- ... T l ..., lffi ..... ~ c-:,
-
Save my notes
Search
■
Search textbook...
---- ·
Q Google search on "Diabetes mellitus (type 2): diagnosis"
Links
QS
You are reviewing a 24-year-old man who has recently been diagnosed
with type 1 diabetes mellitus. He has no comorbidities and works as an
accountant. What HbA1 c target should he aim for initially?
42 mm ol/mol
45 mmol/mol
48 mmol/mol
50 mmol/ mol
52 mmol/mol
Submit answer
Reference ranges v
45 mmol/mol
48 mmol/ mol
50 mmol/mol
52 mmol/ mol
Next question )
Type of insulin
• offer multiple daily injection basal-bolus insulin regimens, rather
than twice-daily mixed insulin regimens, as the insulin injection
regimen of choice for all adults
• twice-daily insulin detemir is the regime of choice. Once-daily
insulin glargine or insulin detemir is an alternative
• offer rapid-acting insulin analogues injected before meals, rather
than rapid-acting soluble human or animal insulins, for mealtime
insulin replacement for adults with type 1 diabetes
Metformin
• NICE recommend considering adding metformin if the BMI >= 25
kg/m 2
Q6
You are called to see a 34 year-old man in the late afternoon while you
are on-call. He suffers with type 1 diabetes mellitus and was admitted
after being diagnosed with diabetic ketoacidosis. He has been treated
with a fixed-rate insulin infusion with potassium replacement. He
usually takes Lantus glargine and Novorapid insulins, but the nurses
have not been administering these while he has been on his insulin
infusion. His latest arterial blood gas is shown:
pH 7.37
p02 11 .9kPa
Bicarbonate 26 mmol/L
Give Novorapid insulin, then stop insulin Infusion with next meal
Give Lantus glargine, then stop insulin infusion with next meal
Submit answer
Reference ranges v
I Give Lantus glarglne, then stop Insulin Infusion with next meal
Next question )
Diabetic ketoacidosis
Features
• abdominal pain
• polyuria, polydipsia, dehydration
• Kussmaul respiration (deep hyperventilation)
• Acetone-smelling breath ('pear drops' smell)
Diagnostic criteria
Fluid Volume
3.5-5.5 40
Hypoglycaemia
Metabolic alkalosis
Hyponatraemia
Hyperkalaemia
Submit answer
Reference ranges v
Metabolic alkalosis
Hyponatraemia
Hyperkalaemia
Next question )
Next question )
Next question )
B I l!!J A ... ·- 2
·- =
1- - - ..... C-:)
Save my notes l
Q8 fc:::J
Glycosuria +++
Submit answer
Reference ranges v
Symptomatic patient with fasting glucose 6.8 mmol/L on two
occasions
Glycosuria +++
Next question )
Q9 Jc::l
Hypercalcaemia
Submit answer
Reference ranges v
I Hypokalaemic metabolic alkalosis
Next question )
Localisation tests
CRH stimulation
• if pituitary source then cortisol rises
• if ectopic/ adrenal then no change in cortisol
MRI adrenals
Submit answer
Reference ranges v
Phenoxybenzamine suppression test
Basics
• bilateral in 10%
• malignant in 10%
• extra-adrenal in 10% (most common site = organ of Zuckerkandl,
adjacent to the bifurcation of the aorta)
Tests
• 24 hr urinary collection of metanephrines (sensitivity 97%*)
• this has replaced a 24 hr urinary collection of catecholamines
(sensitivity 86%)
Next uestion )
Q11
Urinary catecholamines
Serum calcium
Serum prolactin
Submit answer
Reference ranges v
Urinary catecholamines
Serum calcium
Serum prolactin
Next question )
Multiple endocrine neoplasia
The table below summarises the three main types of multiple endocrine
neoplasia (MEN). MEN is inherited as an autosomal dominant disorder.
Parathyroid (95%):
hyperpa rathyro id ism 2 P's 1p
due to parathyroid
hyperplasia Parathyroid (60%) Phaeochromocytom,
Pituitary (70%) Phaeochromocytoma
Pancreas (50%): e.g. Marfanoid body
insulinoma, habitus
gastrinoma (leading Neuromas
to recurrent peptic
ulceration)
Most common
presentation =
hypercalcaemia
MEN type 1
Pancreatic tumours
(e.g. gastrinoma,
insulinoma)
Pituitary tumours
(e .g. prolactinoma)
Primary
hyperparathyroidism
Phaeochromocytoma
Marfanoid body habitus
Medullary thyroid
cancer
Neuromas
RET oncogene
C Passmed'C.ne com
Q12
You review a 47-year-old man one year after he was diagnosed with
prediabetes. Last year he had a HbA 1 c taken after being diagnosed as
having hypertension. This was recorded as being 43 mmol/ mol (6.1%).
His most recent blood test is recorded as being 45 mmol/mol (6.3%)
despite the patient reporting that he has changed his diet as instructed
and exercising three times a week. His body mass index (BMI) today is
26.5 kg/m 2 • Last year it was 27.5kg/m 2 . What is the most appropriate
course of action?
Start metformin
Start pioglitazone
Start orlitstat
Submit answer
Reference ranges v
Start metformin
Start pioglitazone
Start orlitstat
116 11
f • Discuss (7) Improve
Next question )
Terminology
• Diabetes UK currently recommend using the term prediabetes
when talking to patients and impaired glucose regulation when
talkina to other healthcare orofessionals
• research has shown that the term 'prediabetes' has the most
impact and is most easily understood
" ~~
°'
gV.OM 6 1~ l> l'M'<CL1
HbA'1e;
1e
ea 41
.~
\;Jiu..~
mmotnnot
)
~:othilor
:>-= d mmoL'mot
(6~ )
Management
• lifestyle modification: weight loss, increased exercise, change in
diet
• at least yearly follow-up with blood tests is recommended
• NICE recommend metformin for adults at high risk 'whose blood
glucose measure (fasting plasma glucose or HbA 1c) shows they
are still progressing towards type 2 diabetes, despite their
participation in an intensive lifestyle-change programme'
Impaired fasting glucose and impaired glucose tolerance
Definitions
• a fasting glucose greater than or equal to 6.1 but less than 7.0
mmol/ 1implies impaired fasting glucose (IFG)
• impaired glucose tolerance (IGT) is defined as fasting plasma
glucose less than 7.0 mmol/1 and OGTT 2-hour value greater than
or equal to 7.8 mmol/ 1but less than 11 .1 mmol/ 1
• people with IFG should then be offered an oral glucose tolerance
test to rule out a diagnosis of diabetes. A result below 11 .1
mmol/ 1but above 7.8 mmol/1 indicates that the person doesn't
have diabetes but does have IGT
Next question )
B /
Save my notes
Search
Q13
LH 3 .1 mu/I (3-10)
Klinefelter's syndrome
Kallman's syndrome
Submit answer
Reference ranges v
Acute lymphoblastic leukaemia
Kallman's syndrome
The LH and FSH levels are inappropriately low-normal given the low
testosterone concentration, which points towards a diagnosis of
hypogonadotrophic hypogonadism. In Klinefelter's syndrome the LH
and FSH levels are raised
Next question )
I
Next question )
Kallmann's syndrome
Features
• 'delayed puberty'
• hypogonadism, cryptorchidism
• anosmia
• sex hormone levels are low
• LH, FSH levels are inappropriately low/normal
• patients are typically of normal or above average height
Next question)
B I ~
Save my notes
0 Q14
Hypoglycaemia
Weight gain
Submit answer
Reference ranges v
Increased blood pressure
Weight gain
Next question )
9' •• • Discuss (3) Improve
SGLT-2 inhibitors
Patients taking SGLT-2 drugs often lose weight, which can be beneficial
in type 2 diabetes mellitus.
B I A ... ·-
·- 1-
~
- - ...
Save my notes
Q15
Thyroid carcinoma
MEN type 2A
MEN type 1
MEN type 2B
Pheochromocytoma
Submit answer
Reference ranges v
MEN type 2A
MEN type 1
MEN type 2B
Pheochromocytoma
Next question )
Q16
Type 2 diabetes
Rheumatoid arthritis
Recurrent thrush
Submit answer
Reference ranges v
Chronic obstructive pulmonary disease
Type 2 diabetes
Rheumatoid arthritis
Recurrent thrush
Lansoprazole
Amitriptyline
Metoclopramide
Cyclizine
Submit answer
Reference ranges v
Lansoprazole
Amitriptyline
Metoclopramide
Cyclizine
Next question )
Diabetic neuropathy
Diabetes typically leads to sensory loss and not motor loss in peripheral
neuropathy. Painful diabetic neuropathy is a common problem in
clinical practice.
Gastroparesis
• symptoms include erratic blood glucose control, bloating and
vomiting
• management options include metoclopramide, domperidone or
erythromycin (prokinetic agents)
Chronic diarrhoea
• often occurs at night
K+ 5.4 mmol/1
Creatinine 42 umol/1
You arrange for a random cortisol test however whilst awaiting the
result he becomes unresponsive. In addition to giving intravenous
steroid and fluid, what test is it imperative to check first given the likely
diagnosis?
Serum calcium
ECG
Arterial pH
Prolactin
Glucose
Submit answer
I
CamScanner -! W~ :.i.;,-~\
I Glucose l
's disease. The
This que stio n is alluding to a diag nos is of Addison
low bloo d pressure
auto imm une history, raised potassium, fatig ue and
are all clues to this .
ing hypoglycaemia
Patients with Addison 's disease are prone to develop
ids. Given the
due to loss of the gluc oge nic effe ct of gluc oco rtico
checked.
sudden deterioration in GCS, a glucose level mus t be
Neurological
• syncope
• con fusi on
• lethargy
• convulsions
Haemodynamic
• hypotension
• hypothermia
Biochemical
• hyponatraemia
• hyperkalaernia
• hypoglycaemia
'
Next question )
Addisonian crisis
Causes
• sepsis or surgery causing an acute exacerbation of chronic
insufficiency (Addison's, Hypopituitarism)
• adrenal haemorrhage eg Waterhouse-Friderichsen syndrome
(fulminant meningococcemia)
• steroid withdrawal
Management
• hydrocortisone 100 mg im or iv
• 1 litre normal saline infused over 30-60 mins or with dextrose if
hypoglycaemic
• continue hydrocortisone 6 hourly until the patient is stable. No
fludrocortisone is required because high cortisol exerts weak
mineralocorticoid action
• oral replacement may begin after 24 hours and be reduced to
maintenance over 3-4 days
Next question )
C Q19 Jr::J
A 23-year-old woman presents for review. She has not had a normal
period for around 8 months now. A recent pregnancy test was negative.
Blood tests are ordered:
Prolactinoma
Addison's disease
Excessive exercise
Submit answer
Reference ranges v
Addison's disease
Excessive exercise
Next question )
Amenorrhoea
Amenorrhoea may be divided into prima ry (failure to start mense s by
the age of 16 years) or secondary (cessation of established, regular
menstruation for 6 month s or longer).
Initial investigations
• exclude pregnancy with urinary or serum bHCG
• gonadotrophins: low levels indica te a hypot halam ic cause where
as raised levels sugge st an ovarian problem (e.g. Premature
ovarian failure)
• prolactin
• androgen levels: raised levels may be seen in PCOS
• oestradiol
• thyroid functio n tests
Metformin
Losartan
Clopidogrel
Gliclazide
Simvastatin
Submit answer
Reference ranges v
Losartan
Clopidogrel
Gliclazide
Simvastatin
Next question )
Next question )
Sulfonylureas
B I l!!J A ...,
Q21
Xanthomata
Pruritus
Pretibial myxoedema
Eczema
Submit answer
Reference ranges v
I _ a_ ~ ~ - ~
Pretibial myxo_e_d_em
Eczema
is
For the purposes of postg radu ate exam s pretibial myxo edem a
of it
associated with thyrotoxicosis. There are however case repo rts
pittin g
been foun d in hypothyroid patients, especially the diffu se non-
variety
Submit answer
Increases stimulation of insulin secretion by pancreatic 8-cells and
decreases hepatic clearance of insulin
Ca mSc.an,ner ~ '....,.i;lo-" ~ ~ I
Importance: 50
Option 2 best desc ribes the mech anism of actio n of SGLT-2 inhibitors
2
(dap aglifl ozin, cana gliflo zin).[ 1
Optio n 3 best desc ribes the mech anism of actio n of DPP-4 inhibitors
3
( sitag liptin , vilda glipti n). [ ]
Optio n 4 best desc ribes the mech anism of actio n of metf ormi n, an
insul in sens itiser.[4]
es
Option 5 best desc ribes the mech anism of actio n of thiazolidinedion
4
(piog litazo ne), also an insul in sensitiser,[ 1
Acco rding to NICE, metf ormi n should be the first drug treat ment of
choic e for thos e peop le who can tolera te it. The second medication, to
be given in comb inatio n with metf ormi n, can be any one of the four
types of oral hypo glyca emic agents listed_[sJ
Q23
Hyperthyroidism
Retroperitoneal fibrosis
Ascites
Atrial fibrillation
Photosensitivity
Submit answer
Reference ranges v
Retroperitoneal fibrosis
Ascites
Atrial fibrillation
Photosensitivity
Next question )
Next question )
Riedel's thyroiditis
Hypothyroidism Hyperthyrodlsm
H sh tmo10'$
thyroid tlS Subaculti thyrokf
(d Ou rv n' )
Ri
--
rs lhyroid1t11
Postpartum thYTOkfius
Am:.odarone
Gr ves' dis a
~ .......
Next question )
B J I!!! A • ·- 2:
1- - - ....
Q24
Mirabegron
Doxazosin
Surgical repair
Submit answer
Reference ranges v
Mirabegron
Doxazosin
Surgical repair
ntinence.
Surgery is indicated in stress as opposed to urge inco
[ ,. •• tt Discuss Improve ]
Urinary incontinence
Classification
to detr uso r
• overactive bladder (OAB)/urge incontinence: due
overactivity
coughing or
• stress incontinence: leaking sma ll amo unts when
laughing
• mixed incontinence: both urge and stress
tion, e.g. due
• overflow incontinence: due to bladder out let obstruc
to prostate enlargement
Initial investigation
• bladder diaries should be compl eted for a minim um of 3 days
• vaginal examination to exclude pelvic organ prolapse and ability to
initiate voluntary contra ction of pelvic floor muscl es ('Kegel'
exercises)
• urine dipstic k and culture
• urodynamic studies
Save my notes
Q25
th centre with
An 18-year-old man presents to the nurse at the local heal
has vague
a third episode of balanitis over the past 3 mon ths. He also
diag nose d with
sym ptom s of tiredness. His fathe r and gran dfath er were
He is slim with a
type 1 diabetes and take a basal-bolus insulin regimen.
ia on urine
body mas s index of 22 kg/m . He is note d to have glycosur
2
K+ 3.9 mmo l/ 1
Creatinine 91 µmol/ 1
Renal glycosuria
Type 1 diabetes
Type 2 diabetes
Reference ranges v
CamScanne r ➔ •..;~ L - ~1
Maturity onset diabetes of the young (MODY)
Renal glycosuria
Type 1 diabetes
Type 2 diabetes
Next question )
MODY
MODY3
• 60% of cases
• due to a defect in the HNF-1 alpha gene
• is associated with an increased risk of HCC
MODY2
• 20% of cases
• due to a defect in the glucokinase gene
MODY 5
• rare
• due to a defect in the HNF-1 beta gene
• liver and renal cysts
Features of MODY
• typically develops in patients < 25 years
• a family history of early onset diabetes is often present
• ketosis is not a feature at presentation
• patients with the most common form are very sensitive to
sulfonylureas, insulin is not usually necessary
Q26
Submit answer
Reference ranges v
( Anaplastic t hyroid cancer
---------
Next question )
Thyroid cancer
Main points
Type Percentage
Follicular 20%
Main points
Type Percentage
Follicular 20%
Beta-cell hyperplasia
Beta-cell atrophy
Submit answer
Reference ranges v
I Hepatic insulin resistance
Next question )
Terminology
• Diabetes UK currently recommend using the term prediabetes
when talking to patients and impaired glucose regulation when
talking to other healthcare professionals
• research has shown that the term 'prediabetes' has the most
impact and is most easily understood
1 c Inc ~noitc
tnrc:shol:1 IOf b::'es
Management
• lifestyle modification: weight loss, increased exercise, change in
diet
• at least yearly follow-up with blood tests is recommended
• NICE recommend metformin for adults at high risk 'whose blood
glucose measure (fasting plasma glucose or HbA 1c) shows they
are still progressing towards type 2 diabetes, despite their
participation in an intensive lifestyle-change programme'
Next question )
B I t, A .... T I""
Save my notes
Search
■
Search textbook...
----- ·
O. Google search on "Prediabetes and impaired glucose regulation"
Links
Q28
Submit answer
Reference ranges v
Q29
Phaeochromocytoma
Cushing's syndrome
Diabetes insipidus
Macroprolactinoma
Acromegaly
Submit answer
Reference ranges v
I Acrom egaly
Acromegaly: features
Features
• coarse facial appearance, spade-like hands, increa se in shoe size
• large tongue, progn athism , interd ental space s
• excessive sweat ing and oily skin: cause d by sweat gland
hyper trophy
• featur es of pituita ry tumou r: hypop ituitar ism, headaches,
bitemp oral hemia nopia
• raised prolac tin in 1 / 3 of cases -+ galact orrhoe a
• 6% of patien ts have MEN-1
Comp licatio ns
• hyper tensio n
• diabet es (>10%)
• cardio myopa thy
• colore ctal cance r
Which of the following is the most likely cause of this patient's skin
rash?
Acne vulgaris
Erythema nodosum
'
Submit answer
Drug-induced adverse effect
Erythema nodosum
References
(1) 4. Du-Thanh A, Kluger N, Bensalleh H, Guillot B. Drug-Induced
Acneiform Eruption. American Journal of Clinical Dermatology.
2011 ;12(4):233-245.
Corticosteroids: side-effects
Glucocorticoid side-effects
• endocrine: impaired glucose regulation, increased appetite/ weight
gain, hirsutism, hyperlipidaemia
• Cushing's syndrome: moon face, buffalo hump, striae
• musculoskeletal: osteoporosis, proximal myopathy, avascular
necrosis of the femoral head
• immunosuppression: increased susceptibility to severe infection,
reactivation of tuberculosis
• psychiatric: insomnia, mania, depression, psychosis
• gastrointestinal: peptic ulceration, acute pancreatitis
• ophthalmic: glaucoma, cataracts
• dermatological: acne
• suppression of growth in children
• intracranial hypertension
• neutrophilia
Mineralocorticoid side-effects
• fluid retention
• hypertension
Q31
Photosensitivity
Thrombocytopaenia
Myalgia
Peripheral oedema
Hyponatraemia
Submit answer
Reference ranges v
I Peripheral oedema
Hyponatraemia
Thiazolidinediones
Adverse effects
• weight gain
• liver impairment: monitor LFTs
• fluid retention - therefore contrain dicated in heart failure. The risk
of fluid retention is increased if the patient also takes insulin
• recent studies have indicated an increased risk of fracture s
• bladder cancer: recent studies have shown an increased risk of
bladder cancer in patients taking pioglitaz one (hazard ratio 2.64)
A 55-year-old taxi driver with type 2 diabetes mellitus comes for review.
When he was diagnosed 12 months ago he was started on metformin
and the dose was titrated up. His IFCC-HbA 1 cone year ago was 75
mmol/mol (DCCT-HbA 1 c 9%) and is now 69 mmol/mol (8.5%). His body
mass index is 33 kg/m 2 . What is the most appropriate next step in
management?
Add exenatide
Add sitagliptin
Add glipizide
Add insulin
Submit answer
Reference ranges v
Add sitagliptin
Add glipizide
Add insulin
Next question )
Dietary advice
• encourage high fibre, low glycaemic index sources of
carbohydrates
• include low-fat dairy products and oily fish
• control the intake of foods containing saturated fats and trans
fatty acids
• limited substitution of sucrose-containing foods for other
carbohydrates is allowable, but care should be taken to avoid
excess energy intake
• discourage use of foods marketed specifically at people with
diabetes
• initial target weight loss in an overweight person is 5-10%
HbA1c targets
HbA1c
Management of T2DM target
Lifestyle 48
mmol/mol
(6.5%)
Lifestyle+ metformin 48
mmol/mol
(6.5%)
Practical examples
• a patient is newly diagnosed with HbA 1c and wants to try lifestyle
treatment first. You agree a target of 48 mmol/mol (6.5%)
• you review a patient 6 months after starting metformin. His
HbA 1c is 51 mmol/mol (6.8%). You increase his metformin from
500mg bd to 500mg tds and reinforce lifestyle factors
Mettorrmn Glitpln
OR
Sulfonylurea
OR
HbA1c > 58 mmol/mol (7.5%) PloglltaZone
Metformfn
i 1- gltpun
HbA1c > 58 mmolimol (7 5%)
OR I
Metrormln .., su1rony1urea
OR
Metformin • pioglrtazone Ghplln + p1oghtazone
OR OR
Metformln + SGLT-2 Inhibitor Gllptm + sulfonylurea
OR
Plogotazone + sulfonylUrea
HbA1c > 58 mmoL1mol (7 5~o)
i
Metform1n + sulfonylurea -+- GLP-1 muneuc 0 Passmed,cme corn
Drug treatment
Tolerates metformin:
• metformin is still first-line and should be offered if the HbA 1c
rises to 48 mmol/mol (6.5%)* on lifestyle interventions
• if the HbA1c has risen to 58 mmol/mol (7.5%) then a second drug
should be added from the following list:
• - sulfonylurea
• - gliptin
• - pioglitazone
• - SGLT-2 inhibitor
• if despite this the HbA1c rises to, or remains above 58 mmol/mol
(7.5%) then triple therapy with one of the following combinations
should be offered:
• - metformin + gliptin + sulfonylurea
• - metformin + pioglitazone + sulfonylurea
• - metformin + sulfonylurea + SGLT-2 inhibitor
• if despite this the HbA 1c rises to, or remains above 58 mmol/mol
(7.5%) then triple therapy with one of the following combinations
should be offered:
• -. metformin + gliptin + sulfonylurea
• -. metf ormin + pioglitazone + sulfonylurea
• -. metformin + sulfonylurea + SGLT-2 inhibitor
• -. metformin + pioglitazone + SGLT-2 inhibitor
• -. OR insulin therapy should be considered
Practical examples
• you review an established type 2 diabetic on maximum dose
metformin. Her HbA1 c is 55 mmol/mol (7.2%). You do not add
another drug as she has not reached the threshold of 58
mmol/ mol (7.5%)
• a type 2 diabetic is found to have a HbA 1c of 62 mmol/ mol (7.8%)
at annual review. They are currently on maximum dose metformin.
You elect to add a sulfonylurea
Cannot tolerate metformin or contraindicated
• if the HbA1c rises to 48 mmol/ mol (6.5%)* on lifestyle
interventions, consid er one of the followi ng:
• -- sulfonylurea
• -- gliptin
• _. pioglitazone
• if the HbA1c has risen to 58 mmol/ mol (7.5%) then a one of the
followi ng combin ations should be used:
• -- gliptin + pioglitazone
• -- gliptin + sulfonylurea
• -- pioglitazone + sulfonylurea
• if despite this the HbA 1c rises to, or remains above 58 mmol/ mol
(7.5%) then consid er insulin therapy
Starting insulin
• metfor min should be continued. In terms of other drugs NICE
advice: 'Review the continu ed need for other blood glucos e lowerin g
therapies '
• NICE recommend starting with human NPH insulin (isopha ne,
intermediate acting) taken at bed-tim e or twice daily accord ing to
need
Blood pressure
• target is< 140/ 80 mmHg (or < 130/ 80 mmHg if end-organ
damage is present)
• ACE inhibitors are first-line
Anti platelets
• should not be offered unless a patient has existing cardiov ascula r
disease
Atorvaslatin 20mg od
Ator,rastatin IOmg od
d ron-HDL l'..at noc ,_,, by .,,. ~
CCNidlf n-.ig '4> IOIOmg
*this is a bit confusing because isn't the diagnostic criteria for T2DM
HbA1c 48 mmol/mol (6.5%)? So shouldn't all patients be offered
metformin at diagnosis? Our interpretation of this is that some patients
upon diagnosis will elect to try lifestyle measures, which may reduce
their HbA1c below this level. If it then rises to the diagnostic threshold
again metformin should be offered
Save my notes
Q33
Oral doxycycline
Submit answer
Reference ranges v
I Oral ofloxac in + metronidazole
at 22 ,, 30
at f
1 tt Discuss (2) Improve
Causative organisms
• Chlamydia trachomatis
CamScanner -! W~ :.i.;,-~\
Peaiures
• lower abdominal pain
• fever
• deep dyspareunia
• dysuria and menstru al irregularities may occur
• vaginal or cervical discharg e
• cervical excitation
Investigation
• a pregnancy test should be done to exclude an ectopic pregnan cy
• high vaginal swab
o these are often negative
• screen for Chlamyd ia and Gonorrhoea
Manage ment
• due to the difficulty in making an accurate diagnosis, and the
potential complica tions of untreate d PIO, consens us guidelin es
recomm end having a low threshol d for treatme nt
• oral ofloxacin + oral metronid azole or intramus cular ceftriaxo ne +
oral doxycycline + oral metronid azole
• RCOG guidelines suggest that in m ild cases of PIO intrauter ine
contraceptive devices may be left in. The more recent BASHH
guidelines suggest that the evidence is limited but that 'Remova l
of the IUD should be considered and may be associat ed with better
short term clinical outcome s'
Complic ations
• perihepatitis (Fitz-Hugh Curtis Syndrome)
0 occurs in around 10% of cases
Glucokinase
HNF-1 alpha
HNF-4 alpha
HNF-1 beta
IPF-1
Submit answer
Reference ranges v
Q34 ,c
Glucokinase
HNF-1 alpha
HNF-4 alpha
HNF-1 beta
IPF-1
116 11
f • Discuss (2) Improve
Next question )
MODY
rised by the
Matu rity-o nset diabetes of the youn g (MODY) is char acte
years old. It is
deve lopm ent of type 2 diab etes mell itus in patie nts < 25
. Over six
typic ally inhe rited as an auto som al dom inan t cond ition
as lead ing to
diffe rent gene tic muta tions have so far been iden tified
MODY.
MOD Y3
• 60% of case s
• due to a defe ct in the HNF-1 alpha gene
• is asso ciate d with an increased risk of HCC
MODY2
• 20% of case s
• due to a defe ct in the gluc okin ase gene
MODY 5
• rare
• due to a defe ct in the HNF-1 beta gene
• liver and rena l cyst s
Features of MODY
• typic ally develops in patie nts < 25 years
• a fami ly histo ry of early onse t diab etes is ofte n pres ent
• keto sis is not a featu re at pres enta tion
to
• patie nts with the mos t com mon form are very sens itive
sulfo nylu reas, insu lin is not usua lly nece ssar y
Hypothyroidism
Thyroid malignancy
Agranulocytosis
Oesophagitis
Submit answer
Reference ranges v
Q35
I Hypothyroidism
Thyroid malignancy
Agranulocytosis
Oesophagitis
Next question )
Graves• disease: management
ATD titration
• carbimazole is started at 40mg and reduced gradually to maintain
euthyroidism
• typically continued for 12-18 months
• patients following an ATD titration regime have been shown to
suffer fewer side-effects than those on a block-and-replace
regime
Block-and-replace
• carbimazole is started at 40mg
• thyroxine is added when the patient is euthyroid
• treatment typically lasts for 6-9 months
Radioiodine treatment
• contraindications include pregnancy (should be avoided for 4-6
months following treatment) and age< 16 years. Thyroid eye
disease is a relative contraindication, as it may worsen the
condition
• the proportion of patients who become hypothyroid depends on
the dose given, but as a rule the majority of patient will require
thyroxine supplementation after 5 years
Next question )
Q36
Submit answer
Reference ranges v
The major adverse effect is flu-like symptoms
Next question )
mim etic.
Exenatide is an example of a gluca gon- like peptide-1 (GLP-1)
tion.
These drugs increase insulin secretion and inhib it gluca gon secre
ally
One of the majo r advances of GLP-1 mim etics is that they typic
insulin,
result in weig ht loss, in cont rast to man y med icatio ns such as
in
sulfonylureas and thiaz olidin edion es. They are some time s used
comb inatio n with insulin in T2DM to minim ise weig ht gain.
minu tes
Exenatide mus t be given by subc utan eous injec tion withi n 60
after a
before the morn ing and evening mea ls. It shou ld not be given
meal.
the main
Liraglutide is the othe r GLP-1 mim etic curre ntly available. One
be given
advantages of liraglutide over exenatide is that it only needs to
once a day.
n and a
Both exenatide and lirag lutide may be comb ined with metf ormi
used
sulfonylurea. Standard release exenatide is also licensed to be
for a
with basal insulin alone or with metf ormi n. Please see the BNF
more comp lete list of licensed indication s.
occu patio nal impli catio ns or weig ht Joss woul d bene fit othe r
comorbidities.
NICE like patients to have achieved a 11 mmol/mol (1 %) reduction in
HbA 1c and 3% weight loss after 6 months to justify the ongoing
prescription of GLP-1 mimetics.
Key points
• oral preparation
• trials to date show that the drugs are relatively well tolerated with
no increased incidence of hypoglycaemia
• do not cause weight gain
Next question )
Save my notes
Search
Q37
Leptin antagonist
Submit answer
Reference ranges v
Pancreatic lipase inhibitor
I
Next question )
Next question )
Next question )
Save my notes
Q38
Sitagliptin
Pioglitazone
Gliclazide
Exenatide
Metformin
Submit answer
Reference ranges v
Pioglitazone
Gliclazide
Exenatide
Metformin
9' 11
f tt Discuss (2) Improve
A 45-year-old woman is investigated for weight gain. She had had been
unwell for around four months and described a combination of
symptoms including depression, facial male-pattern hair growth and
reduced libido. During the work-up she was found to be hypertensive
with a blood pressure of 170/ 100 mmHg. Which one of the following
tests is most likely to be diagnostic?
Renin:aldosterone levels
Pelvic ultrasound
Submit answer
Reference ranges v
Overnight dexamethasone suppression test
'
Localisation tests
Localisation tests
The first-line localisation is 9am and midn ight plasma ACTH (and
t
corti sol) levels. If ACTH is suppressed then a non-ACTH dependen
cause is likely such as an adrenal adenoma
be
Both low- and high-dose dexamethasone suppression tests may
. These
used to localise the path olog y resulting in Cushing's syndrome
tests may be interpreted as follow s:
Primary hypothyroidism
Addison's disease
Cushing's syndrome
Grave's disease
Bromocriptine
Submit answer
Reference ranges v
Q40
Primary hypothyroidism
Addison's disease
Cushing's syndrome
Grave's disease
Bromocriptine
Next question)
Prolactin and galactorrhoea
Round face
Cognitive impairment
Short stature
Submit answer
Reference ranges v
Round face
Cognitive impairment
Short stature
[ •• •• ,_ Discuss Improve ]
Next question )
Pseudohypoparathyroidism
Bloods
• PTH: high
• calcium: low
• phosphate: high
Features
• short fourth and fifth metacarpals
• short stature
• cognitive impairment
• obesity
• round face
Investigation
• infusion of PTH followed by measurement of urinary phosphate
and cAMP measurement - this can help differentiate between type
I (neither phosphate or cAMP levels rise) and II (cAMP rises but
phosphate levels do not change)
Next question )
01
Adrenal adenoma
Adrenal carcinoma
Pituitary tumour
Submit answer
Reference ranges v
I ~ou_r____
Pituit ary tu_m
Pseudo-Cushing 's
• mimi cs Cushing's
• often due to alcoh ol excess or severe depression
• causes false positi ve dexam ethas one suppr essio n test or 24 hr
urinary free cortis ol
• insulin stress test may be used to differ entia te
Next uestio n )
Q2
Cholecalciferol
Ferrous sulphate
Leveti ra ceta m
Metronidazole
Sertraline
Submit answer
Reference ranges v
I Ferrous sulphate
Levetiracetam
Metronidazole
Sertraline
[ •• •• • Discuss Improve ]
Next question )
Hypothyroidism: management
Key points
• initial starting dose of levothyroxine should be lower in elderly
patients and those with ischaemic heart disease. The BNF
recommends that for patients with cardiac disease, severe
hypothyroidism or patients over 50 years the initial starting dose
should be 25mcg od with dose slowly titrated. Other patients
should be started on a dose of 50-1 00mcg od
• following a change in thyroxine dose thyroid function tests should
be checked after 8-1 2 weeks
• the therapeutic goal is 'normalisation' of the thyroid stimulating
hormone (TSH) level. As the majority of unaffected people have a
TSH value 0.5-2.5 mU/I it is now thought preferable to aim for a
TSH in this range
• women with established hypothyroidism who become pregnant
should have their dose increased 'by at least 25-50 micrograms
levothyroxine'* due to the increased demands of pregnancy. The
TSH should be monitored carefully, aiming for a low-normal value
• there is no evidence to support combination therapy with
levothyroxine and liothyronine
Interactions
• iron, calcium carbonate
o absorption of levothyroxine reduced, give at least 4 hours
apart
47, XO
47, XXV
46, XXV
47, XYY
47, xxo
Submit answer
Reference ranges v
47, XXV
46, XXV
47, XYY
47, xxo
Klinefelter's syndrome
Features
• often taller than average
• lack of secondary sexual characteristics
• small, firm testes
• infertile
• gynaecomastia - increased incidence of breast cancer
• elevated gonadotrophin levels but low testosterone
Next question )
Q4
Submit answer
Hyperkalaemic metabolic alkalosis
,. ,. • Discuss Improve ]
Next question )
I
CamSc.an,ner ~ '....,.i;lo-" ~ ~ I
Q5
K+ 3.9 mmol/ 1
Which one of the following would explain the discrepancy between the
HbA1c and fasting glucose levels?
Conn's syndrome
Sickle-cell anaemia
Submit answer
R81S€0 Cl IOIESLEIOI IEVEI
Sickle-cell anaemia
Next question )
Glycosylated haemoglobin
,, • • t
HbA 1c is generally thought to reflect the blood glucose over the
previous '3 months' although there is some evidence it is weighed more
strongly to glucose levels of the past 2-4 weeks. NICE recommend
'HbA 1c should be checked every 3-6 months until stable, then 6 monthly'.
5 5.5
6 7 .5 42
7 9 .5 53
8 11 .5 64
9 13.5 75
10 15.5
11 17.5
12 19.5
From the above we can see that average plasma glucose = (2 * HbA1c)
-4.5
Q6
Calcium 2.8mmol/l
Phosphate 0.7mmol/l
Parathyroid adenoma
Myeloma
Metastatic cancer
Drug induced
Parathyroid hyperplasia
Submit answer
Parathyroid adenoma
Myeloma
Metastati c cancer
Drug induced
Parathyroid hyperplasia
Next question )
Primary hyperparathyroidism
Associations
• hypertension
• multiple endocrine neoplasia: MEN I and II
Investigations
• raised calcium, low phosphate
• PTH may be raised or (inappropriately, given the raised calcium)
normal
• technetium-MIBI subtraction scan
• pepperpot skull is a characteristic X-ray finding of
hyperparathyroidism
Treatment
• the definitive management is total parathyroidectomy
• conservative management may be offered if the calcium level is
less than 0.25 mmol/L above the upper limit of normal AND the
patient is > 50 years AND there is no evidence of end-organ
damage
• calcimimetic agents such as cinacalcet are sometimes used in
patients who are unsuitable for surgery
Bilateral hand radiographs in a middle-aged woman
demonstrating generalised osteopenia, erosion of the terminal
phalangeal tufts (acro-osteolysis) and subperiosteal resorption
of bone particularly the radial aspects of the 2nd and 3rd middle
phalanges. These changes are consistent with a diagnosis of
hyperparathyroidism.
Q7
Free T 4 26 pmoVI
Hashimoto's thyroiditis
T3-secreting adenoma
De Quervain's thyroiditis
Graves' disease
Submit answer
Graves a1sease
Only around 30% of patients with Graves' disease have eye disease so
the absence of eye signs does not exclude the diagnosis.
16 11
f • Discuss (5) Improve
Next question )
Features
• typical features of thyrotoxicosis
• specific signs limited to Grave's (see below)
Autoantibodies
• TSH receptor stimulating antibodies {90%)
• anti-thyroid peroxidase antibodies (75%)
Q8
Add metformin
Add sitagliptin
Add exenatide
Submit answer
Reference ranges v
A 25-year-old female has type I diabetes. Her HbA 1c is 58 mmol/ L. Her
blood pressure is 126/ 68 mmHg. Her BMI is 28 kg/m 2 • She is using a
basal-bolus regimen which she finds easy to manage. She is not keen to
increase her total insulin dose. Which of the following adjuncts cou ld
you consider to help improve her glycaemic control?
I Add metformin
,---------
Add sitagliptin
Add exenatide
HbA1c
• should be monitored every 3-6 months
• adults should have a target of HbA1c level of 48 mmol/mol (6.5%)
or lower. NICE do however recommend taking into account factors
such as the person's daily activities, aspirations, likelihood of
complications, comorbidities, occupation and history of
hypoglycaemia
Metformin
• NICE recommend considering adding metformin if the BMI >= 25
kg/ m 2
Next question )
-------------------------------
B J !!!J A . . . ·-
· - i::
~ 1- --
Save my notes
Search
Q1
Radioiodine treatment
Thyroidectomy
Propylthiouracil
Carbimazole
Submit answer
Reference ranges v
Thyroidectomy
Propylthiouracil
Carbimazole
Next question )
Pathophysiology
• it is thought to be caused by an autoimmune response against an
autoantigen, possibly the TSH receptor - retro-orbital
inflammation
• the inflammation results in glycosaminoglycan and collagen
deposition in the muscles
Prevention
• smoking is the most important modifiable risk factor for the
development of thyroid eye disease
• radioiodine treatment may increase the inflammatory symptoms
seen in thyroid eye disease. In a recent study of patients with
Graves' disease around 15% developed, or had worsening of, eye
disease. Prednisolone mav help reduce the risk
Features
• the patient may be eu-, hypo- or hyperthyroid at the time of
presentation
• exophthalmos
• conjunctiva! oedema
• optic disc swelling
• ophthalmoplegia
• inability to close the eyelids may lead to sore, dry eyes. If severe
and untreated patients can be at risk of exposure keratopathy
Management
• topical lubricants may be needed to help prevent corneal
inflammation caused by exposure
• steroids
• radiotherapy
• surgery
Rosiglitazone
Metformin
Acarbose
Exenatide
Sitagliptin
Submit answer
Reference ranges v
I Exenatide -~-- ---- ---'
Sitagliptin
Whilst it is well known that insulin resistance and insuff icient B-cell
compensation occur other effect s are also seen in type 2 diabe tes
mellitus (T2DM). In normal physiology an oral glucose load results in a
greater release of insulin than if the same load is given intravenously -
this known as the incretin effect. This effec t is largely mediated by GLP-
1 and is known to be decreased in T2DM .
Liraglutide is the other GLP-1 mimetic currently available. One the main
advantages of liraglutide over exenatide is that it only needs to be given
once a day.
Key points
• oral preparation
• trials to date show that the drugs are relatively well tolerated with
no increased incidence of hypoglycaemia
• do not cause weight gain
Next question )
Save my notes
Q2
Metformin
Glibenclamide
Insulin glargine
Dapagliflozin
Submit answer
Reference ranges v
Glibenclamide
Insulin glargine
Dapagllflozin
Time Target
Next question )
Save my notes
Search
Search textbook...
----■
Q Google search on "Pregnancy: diabetes mellitus''
Links
Ca mSc.an,ner ~ '....,.i;lo-" ~ ~ I
Q3
Submit answer
Reference ranges v
PPAR-gamma receptor agonist
Next question )
Thiazolidinediones
Adverse effects
• weight gain
• liver impairment: monitor LFTs
• fluid retention - therefore contraindicated in heart failure. The risk
of fluid retention is increased if the patient also takes insulin
• recent studies have indicated an increased risk of fractures
• bladder cancer: recent studies have shown an increased risk of
bladder cancer in patients taking pioglitazone (hazard ratio 2.64)
A 29-year-old female who is 7 weeks into her first pregnancy is
investigated for excessive sweating and tremor. Blood tests reveal the
following:
T4 188 nrnol/1
Immediate surgery
Carbimazole
Propylthiouracil
Radioiodine
Submit answer
Reference ranges v
Propylthiouracil
Radioiodine
According to CKS:
[ ,6 •• tt Discuss Improve ]
Next question )
Thyrotoxicosis
CamScanner -! W~ :.i.;,-~\
Graves' disease is the most common cause of thyrotoxicosis in
pregnancy. It is also recognised that activation of the TSH receptor by
HCG may also occur - often termed transient gestational
hyperthyroidism. HCG levels will fall in the second and third trimester
Management
• propylthiouracil has traditionally been the antithyroid drug of
choice
• however, propylthiouracil is associated with an increased risk of
severe hepatic injury
• therefore NICE Clinical Knowledge Summaries advocate the
following : 'Propylthiouracil is used in the first trimester of pregnancy
in place of carbimazole, as the latter drug may be associated with
an increased risk of congenital abnormalities. At the beginning of
the second trimester, the woman should be switched back to
carbimazole'
• maternal free thyroxine levels should be kept in the upper th ird of
the normal reference range to avoid fetal hypothyroidism
• thyrotrophin receptor stimulating antibodies should be checked at
30-36 weeks gestation - helps to determine the risk of neonatal
thyroid problems
• block-and-replace regimes should not be used in pregnancy
• radioiodine therapy is contraindicated
Hypothyroidism
Key points
• thyroxine is safe during pregnancy
• serum thyroid stimulating hormone measured in each trimester
and 6-8 weeks post-partum
• some women require an increased dose of thyroxine during
pregnancy
• breast feeding is safe whilst on thyroxine
Submit answer
Reference ranges v
QS
116 11
f tt Discuss Improve
Next question )
MODY
MODY3
• 60% of cases
• due to a defect in the HNF-1 alpha gene
• is associated with an increased risk of HCC
MODY2
• 20% of cases
• due to a defect in the glucokinase gene
MODY 5
• rare
• due to a defect in the HNF-1 beta gene
• liver and renal cysts
Features of MODY
• typically develops in patients < 25 years
• a family history of early onset diabetes is often present
• ketosis is not a feature at presentation
• patients with the most common form are very sensitive to
sulfonylureas, insulin is not usually necessary
Q6
Hypotension
Depression
Pancreatitis
Submit answer
Reference ranges v
r Hypotension
Depression
Pancreatitis
Next question )
Primary hyperparathyroidism
Associations
• hypertension
• multiple endocrine neoplasia: MEN I and II
Investigations
• raised call cium, low phosphate
• PTH may be raised or (inappropriately, given the raised calcium)
normal
• technetium-MIBI subtraction scan
• pepperpot skull is a characteristic X-ray finding of
hyperparathyroidism
Treatment
• the definitive management is total parathyroidectomy
• conservative management may be offered if the calcium level is
less than 0.25 mmol/L above the upper limit of normal AND the
patient is > 50 years AND there is no evidence of end-organ
damage
• calcimimetic agents such as cinacalcet are sometimes used in
patients who are unsuitable for surgery
Q7
Amitriptyline
lsoniazid
Verapamil
Methyldopa
Spironolactone
Submit answer
Reference ranges v
lsoniazid
Verapamil
Methyl dopa
Spironolactone
All the above drugs may cause gynaecomastia but spironolactone is the
most common cause.
Next question )
Ca mSc.an,ner ~ '....,.i;lo-" ~ ~ I
Gynaecomastia
Causes of gynaecomastia
• physiological: normal in puberty
• syndromes with androgen deficiency: Kallman's, Klinefelter's
• testicular failure: e.g. mumps
• liver disease
• testicular cancer e.g. seminoma secreting hCG
• ectopic tumour secretion
• hyperthyroidism
• haemodialysis
• drugs: see below
Digoxi n
Amlodipine
lvabradine
Bisoprolol
Ramipril
Submit answer
Reference ranges v
( Kam1prn
The answer here is ramipril. The reason behind this is due to its
interference with the renin-angiotensin-aldosterone system, for which
the other medications do not.
Next question )
Primary hyperaldosteronism
Features
• hypertension
• hypokalaemia
o e.g. muscle weakness
o this is a classical feature in exams but studies suggest this
is seen in only 10-40% of patients
• alkalosis
Investigations
• the 2016 Endocrine Society recommend that a plasma
aldosterone/renin ratio is the first-line investigation in suspected
primary hyperaldosteronism
o should show high aldosterone levels alongside low renin
levels (negative feedback due to sodium retention from
aldosterone)
• following this a high-resolution CT abdomen and adrenal vein
sampling is used to differentiate between unilateral and bilateral
sources of aldosterone excess
• Adrenal Venous Sampling (AVS) can be done to identify the gland
secreting excess hormone in primary hyperaldosteronism
Management
• adrenal adenoma: surgery
• bilateral adrenocortical hyperplasia: aldosterone antagonist e.g.
spironolactone
Q9
A 68-year-old man attends his GP surgery for his chronic disease annual
review following on from recent blood tests.
M ale: (135-180)
Hb 142 g/ L
Female: (115 - 160)
Amlodipine
Bendroflumethiazide
Metformin
Prednisolone
Sitagliptin
Submit answer
Metformin
Prednisolone
Sitagliptin
With steroid use, 'left shift' generally does not occur due to increased
production and release of neutrophils from the bone marrow but no
neutrophil consumption.
9' 11
f tt Discuss Improve
Next question )
Next question )
Corticosteroids: side-effects
Glucocorticoid side-effects
• endocrine: impaired glucose regulation, increased appetite/ weight
gain, hirsutism, hyperlipidaemia
• Cushing's syndrome: moon face, buffalo hump, striae
• musculoskeletal: osteoporosis, proximal myopathy, avascular
necrosis of the femoral head
• immunosuppression: increased susceptibility to severe infection,
reactivation of tuberculosis
• psychiatric: insomnia, mania, depression, psychosis
• gastrointestinal: peptic ulceration, acute pancreatitis
• ophthalmic: glaucoma, cataracts
• dermatological: acne
• suppression of growth in children
• intracranial hypertension
• neutrophilia
Mineralocorticoid side-effects
• fluid retention
• hypertension
Next question )
B J I!! A • ·-
·- 1-
~
- - ...
Q10
0 9.2
2 14.2
Start insulin
Start gliclazide
Start metformin
Submit answer
A L~-year-u1u Asian woman wno 1s zo weeKs pregnan1 nas an oral
glucose tolerance test (OGTT). This was requested due to a
combination of her ethnicity and a background of obesity. A recent
ultrasound shows that the fetus is large for dates. The following results
are obtained:
0 9.2
2 14.2
I Start insulin
Start gliclazide
Start metformin
Insulin should be started straight away given the blood glucose levels
and evidence of macrosomia. Aspirin should also be considered as she
is at increased risk of pre-eclampsia.
Next question )
Q11
A 50-year-old female complains that she has put on weight around her
abdomen and th ighs in the last six months. She has a past medical
history of hypertension, high cholesterol, type II diabetes and asthma.
She has not changed her diet or lifestyle during the last six months.
Which of her medications may be contributing to her weight gain?
Atorvastatin
Tolbutamide
Exenatide
Chlortalidone
Saxagliptin
Submit answer
Reference ranges v
Tolbutamide
Exenatide
Chlortalidone
Saxagliptin
BNF
https://bnf.nice.org.uk/ treatment-summary/ type-2-diabetes.html
116 t
11 • Discuss (2) Improve
Sulfonylureas
Next question )
B I A .. 2.=
1- -_ ....
~ c-:::>
Q12
Submit answer
Reference ranges v
rdlll,;lt:dllll::> I::> l,;UIIIIIIUIIIY d::>::>Ul,;ldlt:U VVILII llt:dllllt:lll
She will lose approximately 5kg in weight when she starts treatment
Skin nodules are most often seen around injection sites when modified
release systems are used for delivering GLP-1, such as those used in
weekly exenatide, (microspheres). Pancreatitis features in case reports
associated with the use of GLP-1 agonists, although no consistent link
has been established. Modest weight loss is seen when patients start
degludec / liraglutide against a background of oral agents, but this is
less than that seen for patients who start liraglutide alone.
https://www.evidence.nhs.uk/formulary/bnf/current/6-endocrine-
system/61-drugs-used-in-diabetes/612-antidiabetic-drug s/6123-other-
antidiabetic-drugs
Next question )
Next question )
Key points
• oral preparation
• trials to date show that the drugs are relatively well tolerated with
no increased incidence of hypoglycaem ia
• do not cause weight gain
Next question )
B I
Save my n ates
Search
On examination there was firm irregular mass in the right side of the
anterior triangle of the neck. It was fixed, cold and painless. The mass
moved with swallowing and you note a faint stridor like sound on
inspiration. There was a further 3 irregular lymph nodes of note on
palpation.
Bloods:
Free T4 12 pmol/1
Total T4 99 nmol/1
Papillary
Follicular
Medullary
Anaplastic
Lymphoma
Ana plastic
Lymphoma
Next question )
Thyroid cancer
Main points
Type Percentage
Follicular 20%
Medullary 5% Cancer of parafollicular (C) cells,
secrete calcitonin, part of MEN-2
Further information
Type Notes
Next question )
Save my notes
Q14
Colorectal cancer
Hypertension
Cardiomyopathy
Diabetes mellitus
Pulmonary hypertension
Submit answer
Reference ranges v
I Pulmonary hype_r_te_n_s_io_n_ _ _ _ _~ .
16 11
f • Discuss (4) Improve
Acromegaly: features
Features
• coarse facial appearance, spade-like hands, increase in shoe size
• large tongue, prognathism, interdental spaces
• excessive sweating and oily skin: caused by sweat gland
hypertrophy
• features of pituitary tumour: hypopituitarism, headaches,
bitemporal hemianopia
• raised prolactin in 1/ 3 of cases _... galactorrhoea
• 6% of patients have MEN-1
Complications
• hypertension
• diabetes (>10%)
• cardiomyopathy
• colorectal cancer
Q15 [CJ
pH 7.26
Metformin
Dapagliflozin
Glimepiride
Aspirin
Losartan
Submit answer
Dapagliflozin
Glimepiride
Aspirin
Losartan
SGLT-2 inhibitors
Patients taking SGLT-2 drugs often lose weight, which can be beneficial
in type 2 diabetes mellitus.
Next question )
B I A .... c-:::>
Save my notes
Q16
An example is exenatide
Submit answer
Reference ranges v
Do not cause weight gain
An example is exenatide
Next question )
Liraglutide is the other GLP-1 mimetic currently available. One the main
advantages of liraglutide over exenatide is that it only needs to be given
once a day.
Key points
• oral preparation
• trials to date show that the drugs are relatively well tolerated with
no increased incidence of hypoglycaemia
• do not cause weight gain
BI l!J A ...
Save my notes
Search
Q17 [CJ
Free T4 9 pmol/1
Total T4 67 nmol/1
Hashimoto's thyroiditis
Iodine deficiency
Pendred syndrome
Thyroid agenesis
Atrophic hypothyroidism
Submit answer
Thyroid agenesis
Atrophic hypothyroidism
Next question )
Pendred's syndrome
Vitamin D deficiency
Age of 45
Submit answer
Reference ranges v
Persistent hypercalcaemia over 4 years
Vitamin D deficiency
l Age of 45
From the potential answers offered, the patient's age under 50 is the
only answer that meets the NICE criteria.
Next question )
Primary hyperparathyroidism
Associations
• hypertension
• multiple endocrine neoplasia: MEN I and II
Investigations
• raised calcium, low phosphate
• PTH may be raised or (inappropriately, given the raised ca lcium)
normal
• technetium-MIBI subtraction scan
• pepperpot skull is a characteristic X-ray finding of
hyperparathyroidism
Treatment
• the definitive management is total parathyroidectomy
• conservative management may be offered if the calcium level is
less than 0.25 mmol/L above the upper limit of normal AND the
patient is > 50 years AND there is no evidence of end-organ
damage
• calcimimetic agents such as cinacalcet are sometimes used in
patients who are unsuitable for surgery
Q19
K+ 4 .1 mmol/1
Add pioglitazone
Add exenatide
Add acarbose
Add repaglinide
Add sitagliptin
Submit answer
Add sitagliptin
HbA1c targets
HbA1c
Management of T2DM target
Lifestyle 48
mmol/mol
(6.5%)
Lifestyle+ metformin 48
mmol/mol
(6.5%)
Practical examples
• a patient is newly diagnosed with HbA 1c and wants to try lifestyle
treatment first. You agree a target of 48 mmol/mol (6.5%)
• you review a patient 6 months after starting metformin. His
HbA 1c is 51 mmol/mol (6.8%). You increase his metformin from
500mg bd to 500mg tds and reinforce lifestyle factors
Tolerates metformin:
• metformin is still first-line and should be offered if the HbA 1c
rises to 48 mmol/mol (6.5%)* on lifestyle interventions
• if the HbA 1c has risen to 58 mmol/mol (7.5%) then a second drug
should be added from the following list
• - sulfonylurea
• - gliptin
• - pioglitazone
• - SGLT-2 inhibitor
• if despite this the HbA1 c rises to, or remains above 58 mmol/mol
(7.5%) then triple therapy with one of the following combinations
should be offered:
• - metformin + gliptin + sulfonylurea
• - metformin + pioglitazone + sulfonylurea
• - metformin + sulfonylurea + SGLT-2 inhibitor
• if despite this the HbA1c rises to, or remains above 58 mmol/mol
(7.5%) then triple therapy with one of the following combinations
should be offered:
• - metformin + gliptin + sulfonylurea
• - metformin + pioglitazone + sulfonylurea
• - metformin + sulfonylurea + SGLT-2 inhibitor
• - metformin + pioglitazone + SGLT-2 inhibitor
• - OR insulin therapy should be considered
Practical examples
• you review an established type 2 diabetic on maximum dose
metformin. Her HbA1c is 55 mmol/ mol (7.2%). You do not add
another drug as she has not reached the threshold of 58
mmol/ mol (7.5%)
• a type 2 diabetic is found to have a HbA1c of 62 mmol/mol (7.8%)
at annual review. They are currently on maximum dose metformin.
You elect to add a sulfonylurea
Starting insulin
• metformin should be continued. In terms of other drugs NICE
advice: 'Review the continued need for other blood glucose lowering
therapies'
• NICE recommend starting with human NPH insulin (isophane,
intermediate acting) taken at bed-time or twice daily according to
need
Blood pressure
• target is < 140/ 80 mm Hg (or < 130/ 80 mm Hg if end-organ
damage is present)
• ACE inhibitors are first-line
Antiplatelets
• should not be offered unless a patient has existing cardiovascular
disease
Lipids
• following the 2014 NICE lipid modification guidelines only patients
with a 10-year cardiovascular risk > 10% (using QRISK2) should be
offered a statln. The first-line statin of choice is atorvastatin 20mg
on
- - - - -•- - - - - - - - -•- - - - -
Atorva5taun 20mg od
AtotVastat n IOmg oa
noM-iDL tl • noc fMn DY u .& 1tltn
CCl'l.lid9f lmlli!lg ~ to 80n,g
*this is a bit confusing because isn't the diagnostic criteria for T2DM
HbA1c 48 mmol/ mol (6.5%)? So shouldn't all patients be offered
metformin at diagnosis? Our interpretation of this is that some patients
upon diagnosis will elect to try lifestyle measures, which may reduce
their HbA1c below this level. If it then rises to the diagnostic threshold
again metformin should be offered
Next question )
Save my notes
Q20
Low IQ
Short stature
Submit answer
Reference ranges v
I Low PTH levels
Low IQ
Short stature
Next question )
Hypoparathyroidism
Primary hypoparathyroidism
• decrease PTH secretion
• e.g. secondary to thyroid surgery*
• low calcium, high phosphate
• treated with alfacalcidol
Pseudopseudohypoparathyroidism
• similar phenotype to pseudohypoparathyroidism but normal
biochemistry
Next question )
Save my notes
Search
Submit answer
Reference ranges v
Q21
Next question )
Next question )
Subclinical hyperthyroidism
Causes
• multinodular goitre, particularly in elderly females
• excessive thyroxine may give a similar biochemical picture
Management
• TSH levels often revert to normal - therefore levels must be
persistently low to warrant intervention
• a reasonable treatment option is a therapeutic trial of low-dose
antithyroid agents for approximately 6 months in an effort to
induce a remission
Next question )
B I A ... ·-
·- 2,-: -- . . . T I""
Q22
Free T4 15 pmol/1
Diplopia
Submit answer
Awareness of change in intensity or quality of colour vision
Next question )
Pathophysiology
• it is thought to be caused by an autoimmune response against an
autoantigen, possibly the TSH receptor - retro-orbital
inflammation
• the inflammation results in glycosaminoglycan and collagen
deposition in the muscles
Prevention
• smoking is the most important modifiable risk factor for the
development of thyroid eye disease
• radioiodine treatment may increase the inflammatory symptoms
seen in thyroid eye disease. In a recent study of patients with
Graves' disease around 15% developed, or had worsening of, eye
disease. Prednisolone may help reduce the risk
Management
• topical lubricants may be needed to help prevent corneal
inflammation caused by exposure
• steroids
• radiotherapy
• surgery
Next question )
BI !!f A~
Q23
Submit answer
Reference ranges v
He may be able to apply for a HGV licence if he meets strict criteria
relating to hypoglycaemia
Patients on insulin may now hold a HGV licence if they meet strict
DVLA criteria
Importance: 50
116 11
f • Discuss (2) Improve
Next question )
Until recently people with diabetes who used insulin could not hold a
HGV licence. The DVLA changed the rules in October 2011. The
following standards need to be met (and also apply to patients using
other hypoglycaemic inducing drugs such as sulfonylureas):
• there has not been any severe hypoglycaemic event in the
previous 12 months
• the driver has full hypoglycaemic awareness
• the driver must show adequate control of the condition by regular
blood glucose monitoring*, at least twice daily and at times
relevant to driving
• the driver must demonstrate an understanding of the risks of
hypoglycaemia
• here are no other debarring complications of diabetes
From a practical point of view patients on insulin who want to apply for
a Group 2 (HGV) licence need to complete a VDIAB1 I form.
Next question )
Save my not es
Search
Q24
Which one of the following types of oral steroid has the least amount of
mineralocorticoid activity?
Fludrocortisone
Hydrocortisone
Dexamethasone
Prednisolone
Cortisone
Submit answer
Reference ranges v
Dexamethasone
Prednisolone
Cortisone
116 11
f • Discuss Improve
Next question )
Corticosteroids
Side-effects
The side-effects of corticosteroids are numerous and represent the single greatest limitation on
their usage. Side-effects are more common with systemic and prolonged therapy.
Glucocorticoid side-effects
• endocrine: impaired glucose regulation, increased appetite/weight gain, hirsutism,
hyperlipidaemia
Mineralocorticoid side-effects
• fluid retention
• hypertension
Next question )
B / ~ ~
.t. l ,..
== 1-
i:
=..,.
Save my notes
Q25
Omeprazole
Fluoxetine
Metoclopramide
Cimetidine
Amitriptyline
Submit answer
Reference ranges v
Fluoxetine
Metoclopramide
Cimetidine
Amitriptyline
Importance: 50
Next question )
Next question )
Next question )
Submit answer
Reference ranges v
Increased risk of a stroke
Next question )
Side-effects
• nausea
• breasttenderness
• fluid retention and weight gain
Potential complications
• increased risk of breast cancer: increased by the addition of a
progestogen
• increased risk of endometrial cancer: reduced by the addition of a
progestogen but not eliminated completely. The BNF states that
the additional risk is eliminated if a progestogen is given
continuously
• increased risk of venous thromboembolism: increased by the
addition of a progestogen
• increased risk of stroke
• increased risk of ischaemic heart disease if taken more than 1 O
years after menopause
Breast cancer
• In the Women's Health Initiative (WH I) study there was a relative
risk of 1.26 at 5 years of developing breast cancer
• the increased risk relates to duration of use
• breast cancer incidence is higher in women using combined
preparations compared to oestrogen-only preparations
• the risk of breast cancer begins to decline when HRT is stopped
and by 5 years it reaches the same level as in women who have
never taken H RT
Next question )
Save my notes
Search
Q27
Free T4 14 pmol/ 1
T 4 to T3 conversion disorder
Submit answer
Reference ranges v
Ca mSc.an,ner ~ '....,.i;lo-" ~ ~ I
A 52-year-old woman who was diagnosed as having primary atrophic
hypothyroidism 12 months ago is reviewed following recent thyroid
function tests (TFTs):
TSH 1 2. 5 mU/ 1
Free T4 14 pmol/1
T 4 to T3 conversion disorder
The TSH level is high. This implies that over recent days/ weeks her
body is thyroxine deficient. However, her free T 4 is within normal range.
The most likely explanation is that she started taking the thyroxine
properly just before the blood test. This would correct the thyroxine
level but the TSH takes longer to norma lise.
Next question )
Thyroid function tests
Free
Diagnosis TSH T4 Notes
H s.h1moto's
Rt
--
th~ro;dr IS
ri lhyrotd1US
Subacule lh~ ro1d I.is
(de Ou rv n's)
Amx>darone
Graves' a,s as.e
Next question )
B / l!J
Save my notes
Q28
K+ 3.3 mmol/1
Bicarbonate 26 mmol/1
Creatinine 72 µmol/ 1
Plasma ACTH
Renin:aldosterone ratio
Submit answer
Reference ranges v
Plasma ACTH
Renin:aldosterone ratio
Next question )
Ca mSc.an,ner ~ '....,.i;lo-" ~ ~ I
Q29
Free T4 24 pmol/I
ESR 23 mm/hr
Hashimoto's thyroiditis
Thyroid cancer
De Quervain's thyroiditis
Graves' disease
I
Submit answer
Q29
Free T4 24 pmol/1
ESR 23 mm/ hr
Hashimoto's thyroiditis
Thyroid cancer
De Quervain's thyroiditis
Graves' disease
Submit answer
Reference ranges v
Toxic multinodular goitre
Thyroid cancer
De Quervain's thyroiditis
Graves' disease
Next question )
Next question )
Features
• typica l features of thyrotoxicosis
• specific signs limited to Grave's (see below)
Autoantibodies
• TSH receptor stimulating antibodies (90%)
• anti-thyroid peroxidase antibodies (75%)
Next question )
B / f!!j
Q30 Jc:::1
Raised ESR
Decreased TSH
Submit answer
Reference ranges v
Anti-TSH receptor stimulating antibodies
Decreased TSH
Next question )
Hashimoto's thyroiditis
Features
• features of hypothyroidism
• goitre: firm, non-tender
• anti-thyroid peroxidase and also anti-Tg antibodies
Hypothyroidism Hyperthyrodlsm
Hashrmoto's
thyroidius &lbacute thyroid tis
(da Querva:n's) Graves· disease
0.
..
Next question )
Q1
You review a 68-year-old man who has chron ic obst ructive pulmonary
disease (COPD). Each year he typically has around 7-8 courses of ora l
prednisolone to treat infective exacerbations of his COPD. Which one of
the following adverse effects is linked to long-term steroid use?
Osteomalacia
Enophthalmos
Leucopaenia
Avascular necrosis
Constipation
Submit answer
Reference ranges v
Enophthalmos
Leucopaenia
Avascular necrosis
Constipation
Next question )
Ca mSc.an,ner ~ '....,.i;lo-" ~ ~ I
Q2
Hb 11 .9 g/ dl
Platelets 1 55 * 1o9/ I
Prior to her recent admission the TSH has been within range for the
past two years. Which one of the following new drugs most likely
explains the raised TSH?
Simvastatin
Clopidogrel
Ferrous sulphate
Ramipril
Lansoprazole
Clopidogrel
Ferrous sulphate
Ramipril
Lansoprazole
Next question )
I I _ __ _ _ _._ 1 _ _ • I•
Hypothyroidism: management
Key points
• initial starting dose of levothyroxine should be lower in elderly
patients and those with ischaemic heart disease. The BNF
recommends that for patients with cardiac disease, severe
hypothyroidism or patients over 50 years the initial starting dose
should be 25mcg od with dose slowly titrated. Other patients
should be started on a dose of 50-1 00mcg od
• following a change in thyroxine dose thyroid function tests should
be checked after 8-1 2 weeks
• the therapeutic goal is 'normalisation' of the thyroid stimulating
hormone (TSH) level. As the majority of unaffected people have a
TSH value 0.5-2.5 mU/I it is now thought preferable to aim for a
TSH in this range
• women with established hypothyroidism who become pregnant
should have their dose increased 'by at least 25-50 micrograms
levothyroxine'* due to the increased demands of pregnancy. The
TSH should be monitored carefully, aiming for a low-normal value
• there is no evidence to support combination therapy with
levothyroxine and liothyronine
Interactions
• iron, calcium carbonate
o absorption of levothyroxine reduced, give at least 4 hours
apart
Q3 fc::J
CTabdomen
Submit answer
Reference ranges v
CTabdomen
a6 f
11 tt Discuss (3) Improve
Next question )
CamScanner -! W~ :.i.;,-~\
Primary hyperaldosteronism
Features
• hypertension
• hypokalaemia
o e.g. muscle weakness
o this is a classical feature in exams but studies suggest this
is seen in only 10-40% of patients
• alkalosis
Investigations
• the 2016 Endocrine Society recommend that a plasma
aldosterone/renin ratio is the first-line investigation in suspected
primary hyperaldosteronism
o should show high aldosterone levels alongside low renin
levels (negative feedback due to sodium retention from
aldosterone)
• following this a high-resolution CT abdomen and adrenal vein
sampling is used to differentiate between unilateral and bilateral
sources of aldosterone excess
• Adrenal Venous Sampling (AVS) can be done to identify the gland
secreting excess hormone in primary hyperaldosteronism
Management
• adrenal adenoma: surgery
• bilateral adrenocortical hyperplasia: aldosterone antagonist e.g.
spironolactone
Q4
Submit answer
Reference ranges v
I Concomitant use with insulin
Next question )
Thiazolidinediones
Adverse effects
• weight gain
• liver impairment: monitor LFTs
• fluid retention - therefore contraindicated in heart failu re. The risk
of fluid retention is increased if the patient also takes insulin
• recent studies have indicated an increased risk of fractures
• bladder cancer: recent studies have shown an increased risk of
bladder cancer in patients taking pioglitazone (hazard ratio 2.64)
Next question )
QS
Carbamazepine
Amitriptyline
Pregabalin
Fluoxetine
Sodium valproate
Submit answer
Reference ranges v
Pregabalin
Fluoxetine
Sodium valproate
Next question )
Diabetic neuropathy
Diabetes typically leads to sensory loss and not motor loss in peripheral
neuropathy. Painful diabetic neuropathy is a common problem in
cliniical practice.
Gastroparesis
• symptoms include erratic blood glucose control, bloating and
vomiting
• management options include metoclopramide, domperidone or
erythromycin (prokinetic agents)
Chronic diarrhoea
• often occurs at night
Next question )
B I l!J A y
·- 1-
~
-- y
T IY lffl y
Save my notes
Search
Psychogenic polydipsia
Primary hyperparathyroidism
Pseudohypoparathyroidism
Submit answer
Reference ranges v
I Cranial diabetes insipid_u_s _ _ _ _ ~
A dramatic improvement is seen in the ability of the kidneys to
concentrate urine following the administration of DDAVP. This points
towards a diagnosis of cranial diabetes insipidus
Next question )
Method
• prevent patient drinking water
• ask patient to empty bladder
• hourly urine and plasma osmolalities
Final
Starting urme Urine osm.
plasma osm. osm. post-DDAVP
Next question )
Q7
Clitoromegaly
Acanthosis nigricans
Obesity
Amenorrhoea
Acne
Submit answer
Reference ranges v
Q7
I Clitoromegaly
---------
Acanthosis nigricans
Obesity
Amenorrhoea
Acne
Next question )
Features
• subfertility and infertility
• menstrual disturbances: oligomenorrhea and amenorrhoea
• hirsutism, acne (due to hyperandrogenism)
• obesity
• acanthosis nigricans (due to insulin resistance)
Investigations
• pelvic ultrasound: multiple cysts on the ovaries
• FSH, LH, prolactin, TSH, and testosterone are useful
investigations: raised LH:FSH ratio is a 'classical' feature but is no
longer thought to be useful in diagnosis. Prolactin may be normal
or mildly elevated. Testosterone may be normal or mildly elevated
- however, if markedly raised consider other causes
• check for impaired glucose tolerance
Next question )
'
QB
Biguanide
Sulfonylurea
GLP-1 mimetic
SGLT-2 inhibitor
DPP-4 inhibitor
Submit answer
Reference ranges v
SGLT-2 inhibitor
DPP-4 inhibitor
SGLT-2 inhibitors are often the best option in patients with ischaemic
heart disease and heart failure who have not achieved adequate
glycaemic control on metformin alone.
Next question )
SGLT-2 inhibitors
Patients taking SGLT-2 drugs often lose weight, which can be beneficial
in type 2 diabetes mellitus.
Next question )
B I tj A • ·- J= =• T I• ~• ~ c-::>
Save my notes
Q9
Exogenous thyroxine
Graves' Disease
Hashimoto's thyroiditis
Post-partum thyroiditis
De Quervain's thyroiditis
Submit answer
I Graves' Disease
Hashimoto's thyroiditis
Post-partum thyroiditis
De Quervain's thyroiditis
De Quervain's thyroiditis can present with pain and dysphagia and may
lead to high, normal or low thyroid levels.
Next question )
l 'f CJ\l \.{ UC.:>l l UI I r
Thyrotoxicosis
Management
• propylthiouracil has traditionally been the antithyroid drug of
choice
• however, propylthiouracil is associated with an increased risk of
severe hepatic injury
• therefore NICE Clinical Knowledge Summaries advocate the
following: 'Propylthiouracil is used in the first trimester of pregnancy
in place of carbimazo/e, as the latter drug may be associated with
an increased risk of congenital abnormalities. At the beginning of
the second trimester, the woman should be switched back to
carbimazole'
• maternal free thyroxine level s should be kept in the upper third of
the normal reference range to avoid fetal hypothyroidism
• thyrotrophin receptor stimulating antibodies should be checked at
30-36 weeks gestation - helps to determine the risk of neonatal
thyroid problems
• block-and-replace regimes should not be used in pregnancy
• radioiodine therapy is contraindicated
Hypothyroid ism
Key points
• thyroxine is safe during pregnancy
• serum thyroid stimulating hormone measured in each trimester
and 6-8 weeks post-partum
• some women require an increased dose of thyroxine during
pregnancy
• breast feeding is safe whilst on thyroxine
Next question )
B / !!! T 1-
~ -
-T T IT
Save my notes
Search
■
Search textbook...
---- ·
0. Google search on "Pregnancy: thyroid problems"
Q10
What is the most appropriate first line treatment for his pain?
Tramadol
Topical capsaicin
Codeine
Duloxetine
Naproxen
Submit answer
Reference ranges v
Topical capsaicin
Codeine
Duloxetine
Naproxen
Tramadol is also not the first line treatment but can be used as rescue
therapy for exacerbations.
a6 Rf t9 Discuss Improve
Diabetes typically leads to sensory loss and not motor loss in peripheral
neuropathy. Painful diabetic neuropathy is a common problem in
clinical practice.
Gastroparesis
• symptoms include erratic blood glucose control, bloating and
vomiting
• management options include metoclopramide, domperidone or
erythromycin (prokinetic agents)
Chronic diarrhoea
• often occurs at night
Free T 4 24 pmoVI
Grave's disease
De Quervain's thyroiditis
Hashimoto's thyroiditis
Submit answer
Reference ranges v
Sick thyroid syndrome
De Quervain's thyroiditis
Hashimoto's thyroiditis
Investigations
• thyroid scintigraphy: globally reduced uptake of iodine-131
Management
• usually self-limiting - most patients do not require treatment
• thyroid pain may respond to aspirin or other NSAIDs
• in more severe cases steroids are used, particularly if
hypothyroidism develops
Hypothyroidism .«yperthyrodlsm
H hlfflOto's
thyroid us
__
Subacute thyroid I.ls
(de Ouervam's) ..,
L1th1Um
Submit answer
Reference ranges v
Confirms a diagnosis of Conn's disease
The correct answer is that this is an inconclusive result and the patient
should be further investigated with a short Synacthen test. 9 am
cortisol results between 100-500nmol/l are inconclusive and patients
should be referred to secondary care for further investigation. A result
less than 1 oonmol/I makes Addison's disease or hypoadrenalism likely,
whilst a result >S00nmol/1 makes a diagnosis of Addison's disease very
unlikely.
Simply repeating the test would not be the correct answer unless there
was concern that the test was not taken at the correct time.
Next question )
Next question )
Next question )
B / ~ T I•
Q13
Submit answer
Reference ranges v
I Double hydrocortisone to 40mg mornings and 20mg afternoon
Next question )
Q14
11-hydroxylase deficiency
Diethylstilbestrol deficiency
Oestrogen deficiency
Submit answer
Reference ranges v
11-hydroxylase deficiency
Diethylstilbestrol deficiency
Oestrogen deficiency
'
Next question )
1'4CJU ~UC~l l UI I r
Addison's disease
Features
• lethargy, weakness, anorexia, nausea & vomiting, weight loss, 'salt-
craving'
• hyperpigmentation (especially palmar creases)*, vitiligo, loss of
pubic hair in women, hypotension, hypoglycaemia
• hyponatraemia and hyperkalaemia may be seen
• crisis: collapse, shock, pyrexia
Primary causes
• tuberculosis
• metastases (e.g. bronchial carcinoma)
• meningococcal septicaemia (Waterhouse-Friderichsen syndrome)
• HIV
• antiphospholipid syndrome
Secondary causes
• pituitary disorders (e.g. tumours, irradiation, infiltration)
Next question )
0 Q15
Submit answer
Reference ranges v
Add either pioglitazone, a DPP-4 inhibitor or a SGLT-2 inhibitor
Next question )
HbA1c targets
HbA1c
Management of T2DM target
Lifestyle 48
mmol/mol
(6.5%)
Lifestyle+ metformin 48
mmol/mol
(6.5%)
Practical examples
• a patient is newly diagnosed with HbA1 c and wants to try lifestyle
treatment first. You agree a target of 48 mmol/mol (6.5%)
• you review a patient 6 months after starting metformin. His
HbA 1c is 51 mmol/ mol (6.8%). You increase his metformin from
500mg bd to 500mg tds and reinforce lifestyle factors
Tolerates metformin:
• metformin is still first-line and should be offered if the HbA 1c
rises to 48 mmol/mol (6.5%)* on lifestyle interventions
• if the HbA1c has risen to 58 mmol/mol (7.5%) then a second drug
should be added from the following list:
• - sulfonylurea
• - gliptin
• - pioglitazone
• - SGLT-2 inhibitor
• if despite this the HbA1c rises to, or remains above 58 mmol/ mol
(7.5%) then triple therapy with one of the following combinations
should be offered:
• - metformin + gliptin + sulfonylurea
• - metformin + pioglitazone + sulfonylurea
• - metformin + sulfonylurea + SGLT-2 inhibitor
• - metformin + pioglitazone + SGLT-2 inhibitor
• - OR insulin therapy should be considered
Practical examples
• you review an established type 2 diabetic on maximum dose
metformin. Her HbA1 c Is 55 mmol/ mol (7.2%). You do not add
another drug as she has not reached the threshold of 58
mmol/mol (7.5%)
• a type 2 diabetic is found to have a HbA1c of 62 mmol/mol (7.8%)
at annual review. They are currently on maximum dose metformin.
You elect to add a sulfonylurea
Starting insulin
• metformin should be continued. In terms of other drugs NICE
advice: 'Review the continued need for other blood glucose lowering
therapies'
• NICE recommend starting with human NPH insulin (isophane,
intermediate acting) taken at bed-time or twice daily according to
need
Blood pressure
• target is< 140/ 80 mmHg (or< 130/ 80 mmHg if end-organ
damage is present)
• ACE inhibitors are first-line
Antiplatelets
• should not be offered unless a patient has existing cardiovascular
disease
Lipids
• following the 2014 NICE lipid modification guidelines on ly patients
with a 10-year cardiovascular risk> 10% (using QRISK2) shou ld be
offered a statin. The first-line statin of choice is atorvastatin 20mg
on
Atorva,tat111 20mg od
! non-HOL t1a1 not ~ by ,.,. , thin AtOfVHtat " IOmg od
c0f!Sidsr llhllllg up to 81lm;
"1his is a bit confusing because isn't the diagnostic criteria for T2DM
HbA 1c 48 mmol/ mol (6.5%)? So shouldn't all patients be offered
metformin at diagnosis? Our interpretation of this is that some patients
upon diagnosis will elect to try lifestyle measures, which may reduce
their HbA1c below this level. If it then rises to the diagnostic threshold
again metformin shou ld be offered
Next question )
B I !!J A .... ·-
·- 2=
1- -- 'IP'
01
Hypokalaemia
Hypertension
Metabolic alkalosis
Hypocalciuria
Hypomagnesaemia
Submit answer
Reference ranges v
Hypertension
Metabolic alkalosis
Hypocalciuria
Hypomagnesaemia
9' f
11
• Discuss (3) Improve
Gitelman's syndrome
Features
• normotension
• hypokalaemia
• hypocalciuria
• hypomagnesaemia
• metabolic alkalosis
Q2
Which one of the following is not part of the diagnostic criteria for the
metabolic syndrome?
High triglycerides
Low HDL
High LDL
Central obesity
Hypertension
Submit answer
Reference ranges v
LUVV nuL
High LDL
Central obesity
Hypertension
High LDL levels are not part of the World Health Organization or
International Diabetes Federation diagnostic criteria
[ ,6 •• tt Discuss Improve
Next question )
Metabolic syndrome
ic crite ria
In 1999 the World Health Organization produced diag nost
gluc ose
which required the presence of diabetes mellitus, impaired
AND two of
tolerance, impaired fasti ng gluc ose or insulin resistance,
the follo wing :
• blood pressure: > 140/ 90 mmH g
-den sity
• dyslipidaemia : trigly cerid es: > 1.695 mm ol/L and/ or high
mm ol/L
lipoprotein cholesterol (HDL-C) < 0.9 mm ol/L (male),< 1.0
(female)
ale),
• central obes ity: wais t:hip ratio > 0.90 (male), > 0.85 (fem
and/ or body mas s index > 30 kg/ m2
mg/ min or
• microalbuminuria: urinary albumin excretion ratio > 20
albumin:creatinine ratio > 30 mg/ g
Submit answer
Reference ranges v
Oral glucose tolerance test with growth hormone measurements
Next question )
Next question >
lnsulinoma
Basics
• most common pancreatic endocrine tumour
• 10% malignant, 10% multiple
• of patients with multiple tumours, 50% have MEN-1
Features
• of hypoglycaemia: typically early in morning or just before meal,
e.g. diplopia, weakness etc
• rapid weight gain may be seen
• high insulin, raised proinsulin:insulin ratio
• high C-peptide
Diagnosis
• supervised, prolonged fasting (up to 72 hours)
• CT pancreas
Management
• surgery
• diazoxide and somatostatin if patients are not candidates for
surgery
Next question )
Q4
A 60-year-old man who is known to have lung cancer comes for review.
For the past three weeks he has lost his appetite, has been feeling sick
and generally feels tired. On examination he appears to be mildly
dehydrated. You order some blood tests:
Albumin 40 g/ I
Amlodipine
Simvastatin
Bendroflu methiazide
Aspirin
Lisinopril
Submit answer
Simvastatin
Bendroflumethiazide
Aspirin
Lisinopril
16 .. • Discuss Improve
Next question )
- -.... ......., .....""!II,.....- - --,-- . .,
Hypercalcaemia: causes
**usually normal in this condi tion but hypercalcaemia may occur with
prolonged immob ilisatio n
Pancreatic carcinoma
Colorectal carcinoma
Adrenal carcinoma
Malignant melanoma
Lung cancer
Submit answer
Reference ranges v
Colorectal carcinoma ]
Adrenal carcinoma
Malignant melanoma
Lung cancer
[ ,. ,. - Discuss Improve ]
Next question )
CamSc.an,ner ~ '....,.i;lo-" ~ ~ I
Acromegaly: features
Features
• coarse facial appearance, spade-like hands, increase in shoe size
• large tongue, prognathism, interdental spaces
• excessive sweating and oily skin: caused by sweat gland
hypertrophy
• features of pituitary tumour: hypopituitarism, headaches,
bitemporal hemianopia
• raised prolactin in 1/ 3 of cases - galactorrhoea
• 6% of patients have MEN-1
Complications
• hypertension
• diabetes (>10%)
• cardiomyopathy
• colorectal cancer
Next question )
B I l!J A ... ·- 1-
~
- - ...
Save my notes
Q6
'
Submit answer
Reference ranges v
A 23-year-old woman is diagnosed with Graves' disease. Which one of
the following statements regarding treatment is correct?
Next question )
Graves' disease: ma nag em ent
al
Despite man y trials there is no clea r guid ance on the optim
de titra tion of
man agem ent of Graves' dise ase. Trea tmen t optio ns inclu
k-and-replace
anti-thyroid drugs (ATDs, for exam ple carbimazole), bloc
is ofte n given
regimes, radioiodine trea tmen t and surgery. Propranolol
initially to bloc k adrenergic effe cts
Block-and-replace
• carbimazole is start ed at 40m g
• thyroxine is added when the patie nt is euthyroid
• trea tmen t typic ally lasts for 6-9 mon ths
CamScanne r ➔ •..;~ L - ~1
Q7
Klinefelter's syndrome
Seminoma
Liver disease
Puberty
Hypothyroidism
Submit answer
Reference ranges v
Seminoma
Liver disease
Puberty
Hypothyroidism
Next question )
Gynaecomastia
Next question )
B I ~ A ,..
Save my notes
Q8
A 25-year-old woman presents for her first cervical smear. What is the
most important aetiological factor causing cervical cancer?
Smoking
Submit answer
Reference ranges v
Herpes simplex virus 2
Smoking
Next question )
Cervical cancer
Around 50% of cases of cervical cance r occur in wome n under the age
of 45 years, with incidence rates for cervical cance r in the UK are
highes t in people aged 25-29 years, accord ing to Cancer Research UK. It
may be divided into:
• squam ous cell cancer (80%)
• adenocarcinoma (20%)
Features
• may be detected during routine cervical cance r screen ing
• abnormal vaginal bleeding: postco ital, interm enstru al or
postm enopa usal bleeding
• vaginal discharge
*the strength of this assoc iation is some times debated but a large
study published in the Lancet (2007 Nov 10;370 (9599 ):1609-21)
confir med the link
0 Q9
Urinalysis NAO
6 months
1 month
2 weeks
3 months
4 months
Submit answer
Reference ranges v
13months
4 month s
A more accurate answe r would proba bly be 2 month s but this is not
given as an option . See the explan ation below
Glycosylated haemoglobin
5 5.5
6 7.5 42
7 9.5 53
8 11 .5 64
9 13.5 75
10 15.5
11 17.5
12 19.5
= (2 * HbA 1c)
From the above we can see that average plas ma gluc ose
-4.5
Dynamic pituitary function tests may be used to assess each one of the
following, except:
Cortisol
Prolactin
Growth hormone
Antidiuretic hormone
Submit answer
Reference ranges v
I Antldi uretic hormo ne
Insulin, TRH and LHRH are given to the patien t follow ing which the
serum glucose, cortiso l, growt h hormo ne, TSH, LH and FSH levels are
recorded at regular interva ls. Prolactin levels are also some times
measu red*
*dopa mine antag onist tests using metoc lopram ide may also be used in
the invest igation of hyper prolac tinaem ia. A norma l respo nse is at least
a twofo ld rise in prolactin. A blunte d prolac tin respo nse sugge sts a
prolac tinoma
Next question )
Q11
Obesity
Erratic lifestyle
Submit answer
Reference ranges v
Erratic lifestyle
Next question )
HbA1c targets
CamSc.an,ner ~ '....,.i;lo-" ~ ~ I
Lifestyle or single drug treatme nt
HbA1c
target
Manage ment of T2DM
48
Lifestyle
mmol/ mol
(6.5%)
Lifestyle + metform in 48
mmol/m ol
(6.5%)
Practical examples
• a patient is newly diagnosed with HbA 1 c and wants to try lifestyle
treatme nt first. You agree a target of 48 mmol/m ol (6.5%)
• you review a patient 6 months after starting metform in. His
HbA 1 c is 51 mmol/m ol (6.8%). You increase his metform in from
500mg bd to 500mg tds and reinforce lifestyle factors
Tolerates metformin:
• metformin is still first-line and should be offered if the HbA 1c
rises to 48 mmol/mol (6.5%)* on lifestyle interventions
• if the HbA1c has risen to 58 mmol/ mol (7.5%) then a second drug
should be added from the following list:
• - sulfonylurea
• - gliptin
• - pioglitazone
• - SGLT-2 inhibitor
• if despite this the HbA1 c rises to, or remains above 58 mmo l/mo l
(7.5%) then triple therapy with one of the follow ing comb inatio ns
should be offered:
• -+ metfo rmin + glipti n + sulfo nylur
ea
• -+ metfo rmin + piogl itazo ne + sulfo
nylurea
• -+ metfo rmin + sulfo nylur ea + SGLT
-2 inhib itor
• - metfo rmin + pioglitazone + SGLT-2 inhib itor
• - OR insulin therapy should be considered
weight loss would benefit other signi fican t obesityrelated como rbidit ies
• only continue if there is a reduction of at least 11 mmo l/mo l
[1 .0%] in HbA1c and a weig ht loss of at least 3% of initial body
weig ht in 6 mont hs
Practical examples
• you review an established type 2 diabe tic on maxi mum dose
metfo rmin . Her HbA 1c is 55 mmo l/ mol (7.2%). You do not add
another drug as she has not reached the thres hold of 58
mmo l/ mol (7.5%)
• a type 2 diabetic is found to have a HbA 1c of 62 mmo l/mo l (7.8%)
at annual review. They are currently on maxi mum dose metfo rmin.
You elect to add a sulfonylurea
Cannot tolerate metformin or contraindicated
• if the HbA1crises to 48 mmol /mol (6.5%) * on lifesty le
interventions, consid er one of the follow ing:
• _. sulfonylurea
• _. gliptin
• _. pioglitazone
• if the HbA 1c has risen to 58 mmol / mol (7.5%) then a one of the
follow ing combi nation s should be used:
• _. gliptin + pioglitazone
• -+ gliptin + sulfon ylurea
• _. pioglitazone + sulfonylurea
• if despite this the HbA 1c rises to, or remains above 58 mmol /mol
(7.5%) then consid er insulin therapy
Starting insulin
• metfo rmin should be contin ued. In terms of other drugs NICE
advice : 'Review the contin ued need for other blood glucos e loweri ng
therapies '
• NICE recommend startin g with huma n NPH insulin (isophane,
intermediate acting) taken at bed-tim e or twice daily accor ding to
need
Blood pressure
• target is< 140/8 0 mmHg (or< 130/ 80 mmHg if end-organ
damage is present)
• ACE inhibitors are first-line
Antiplatelet s
• should not be offered unless a patien t has existin g cardio vascu lar
disease
Lipids
• following the 2014 NICE lipid modifica tion guidelines only patients
with a 10-year cardiova scular risk > 10% (using QRISK2) should be
offered a statin. The first-line statin of choice is atorvast atin 20mg
on
Atorvastatln 20mg od
Atorvastatin IOmg od
non-HOL nn noc t by .,. ,o,. tnen
cCtlllder wnng I.IP IO 81),,g
-kthis is a bit confusin g because isn't the diagnos tic criteria for T2DM
HbA 1c 48 mmol/m ol (6.5%)? So shouldn 't all patients be offered
metform in at diagnosis? Our interpret ation of this is that some patients
upon diagnos is will elect to try lifestyle measures, which may reduce
their HbA 1c below this level. If it then rises to the diagnos tic threshol d
again metform ln should be offered
Next question )
B / tj A,..
Save my notes
0 Q12
Submit answer
Reference ranges v
Concordance between identical twins is higher in type 2 diabetes
mellitus than type 1
Next question )
CamSc.an,ner ~ '....,.i;lo-" ~ ~ I
Next question )
Diabetes: pathophysiology
Type 1 OM
• autoimmune disease
• antibodies against beta cells of pancreas
• HLA OR4 > HLA DR3
• various antibodies such as islet-associated antigen (IAA) antibody
and glutamic acid decarboxylase (GAD) antibody are detected in
patients who later go on to develop type 1 OM - their prognostic
significance is not yet clear
Next question )
B I l!!J
Save my notes
Search
■
Search textbook...
---- ·
Q Google search on "Diabetes: pathophysiology"
Q13
Alkalosis
Hypertension
Hypokalaemia
Submit answer
Reference ranges v
Response to treatment with amiloride
Hypertension
Hypokalaemia
•• f
11
• Discuss (1) Improve
Next question )
Liddle's syndrome
Save my notes
Search
Q14
Anaplastic
Lymphoma
Follicular
Medullary
Submit answer
Reference ranges v
Medullary
116 f
11 tt Discuss Improve
The table below summarises the three main types of multiple endocrine
neoplasia (MEN). MEN is inherited as an autosomal dominant disorder.
Parathyroid (95%):
hyperparathyroidism 2 P's 1P
due to parathyroid
hyperplasia Parathyroid (60%) Phaeochromocytomc
Pituitary (70%) Phaeochromocytoma
Pancreas (50%): e.g. Marfanoid body
insulinoma, habitus
gastrinoma (leading Neuromas
to recurrent peptic
ulceration)
Also: adrenal and
thyroid
Most common
presentation =
hypercalcaemia
MEN type 1
P oc, he tumours
(e g gastnnoma
su!inoma)
Pitultary tumours
<• g PfO cunornaJ
Primary
hyperparathyroidrsm
PhaeocPuomocytoma
Marfanoid body habitus
Medullaiy thyroid
ca!lCer
Neuromas
RE T oncogene
Next question )
B J ~ A ...
Q15
Total T4
TSH
Free T4
ESR
Free T3
Submit answer
Reference ranges v
Free T4
ESR
Free T3
Ne xt que stio n )
Thyrotoxicosis: causes and investigation
Causes
• Graves' disease
• toxic nodular goitre
• acute phase of subacute (de Quervain's) thyroiditis
• acute phase of post-partum thyroiditis
• acute phase of Hashimoto's thyroiditis (later results in
hypothyroidism)
• amiodarone therapy
Investigation
• TSH down, T4 and T3 up
• thyroid autoantibodies
• other investigations are not routinely done but includes isotope
scanning
Hypothyroidism Hyperthyrodism
H st11moo's
Chyroidi,:,s &lblacute th~roicl t.s
(de Ouervarn's) Graves' disease
Ill
1O unit / hour
1 unit / hour
2 unit / hour
6 unit / hour
8 unit / hour
Submit answer
Reference ranges v
I 8 unit/ hour
Next question )
Diabetic ketoacidosis
Pathophysiology
• DKA is caused by uncontrolled lipolysis (not proteolysis) which
results in an excess of free fatty acids that are ultimately
converted to ketone bodies
Features
• abdominal pain
• polyuria, polydipsia, dehydration
• Kussmaul respiration (deep hyperventilation)
• Acetone-smelling breath ('pear drops' smell)
Diagnostic criteria
Management
• fluid replacement: most patients with OKA are deplete around 5-8
litres. Isotonic saline is used initially. Please see an example fluid
regime below.
• insulin: an intravenous infusion should be started at 0.1
unit/ kg/ hour. Once blood glucose is < 15 mmol/ 1an infusion of 5%
dextrose should be started
• correction of hypokalaemia
• long-acting insulin should be continued, short-acting insulin
should be stopped
Fluid Volume
Please note that slower infusion may be indicated in young adults (aged
18-25 years) as they are at greater risk of cerebral oedema .
3.5-5.5 40
Next question )
B I = ...
- T I ...
Save my notes
Search
Q17
Hyper- or euthyroid
Hypothyroid
0 Hyperthyroid
Hypo- or euthyroid
Submit answer
Reference ranges v
I Eu-, hypo- or hyperthyro-id~
It is also known that the severity of thyroid eye disease is not related to
the degree of thyrotoxicosis in Graves' disease.
Next question )
Pathophysiology
• it is thought to be caused by an autoimmune response against an
autoantigen, possibly the TSH receptor -- retro-orbital
inflammation
• the inflammation results in glycosaminoglycan and collagen
deposition in the muscles
Prevention
• smoking is the most important modifiable risk factor for the
development of thyroid eye disease
• radioiodine treatment may increase the inflammatory symptoms
seen in thyroid eye disease. In a recent study of patients with
Graves' disease around 15% developed, or had worsening of, eye
disease. Prednisolone may help reduce the risk
Features
• the patient may be eu-, hypo- or hyperthyroid at the time of
presentation
• exophthalmos
• conjunctiva! oedema
• optic disc swelling
• ophthalmoplegia
• inability to close the eyelids may lead to sore, dry eyes. If severe
and untreated patients can be at risk of exposure keratopathy
Management
• topical lubricants may be needed to help prevent corneal
inflammation caused by exposure
• steroids
• radiotherapy
• surgery
Next question )
Q18
Down's syndrome
Kallman's syndrome
Dubin-Johnson syndrome
Turner's syndrome
Klinefelter's syndrome
Submit answer
Reference ranges v
Dubin-Johnson syndrome
Turner's syndrome
Klinefelter's syndrome
Next question )
Kallmann's syndrome
Features
• 'delayed puberty'
• hypogonadism, cryptorchidism
• anosmia
• sex hormone levels are low
• LH, FSH levels are inappropriately low/ normal
• patients are typically of normal or above average height
Cleft lip/ palate and visual/hearing defects are also seen in some
Q19
All patient should have a repeat oral glucose tolerance test every
2 years
Submit answer
Reference ranges v
Patients with impaired glucose tolerance are more likely to develop
diabetes than patients with impaired fasting glycaemia
Terminology
• Diabetes UK currently recommend using the term prediabetes
when talking to patients and impaired glucose regulation when
talking to other healthcare professionals
• research has shown that the term 'prediabetes' has the most
impact and is most easily understood
Identification of patients with prediabetes
• NICE recommend using a validated computer based risk
assessment tool for all adults aged 40 and over, people of South
Asian and Chinese descent aged 25-39, and adults with
conditions that increase the risk of type 2 diabetes
• patients identified at high risk should have a blood sample taken
• a fasting plasma glucose of 6.1-6. 9 mmol/ 1or an HbA1c level of
42-47 mmol/ mol (6 .0-6.4%) indicates high risk
,rediabetH
Normal g~m•c control Diabnn meliilus
.,.'bi\ le ~2....&7 rrm:,lmci 6 ~ -I
1$
or '
C: n~ g\leow 6 1-6 Pm~,
Management
• lifestyle modification: weight loss, increased exercise, change in
diet
• at least yearly follow-up with blood tests is recommended
• NICE recommend metformin for adults at high risk 'whose blood
glucose measure (fasting plasma glucose or HbA 1c) shows they
are still progressing towards type 2 diabetes, despite their
participation in an intensive lifestyle-change programme'
Impaired fasting glucose and impaired glucose tolerance
Definitions
• a fasting glucose greater than or equal to 6.1 but less than 7.0
mmol/1 implies impaired fasting glucose (IFG)
• impaired glucose tolerance (IGT) is defined as fasting plasma
glucose less than 7.0 mmol/1 and OGTT 2-hour value greater than
or equal to 7.8 mmol/1 but less than 11.1 mmol/1
• people with IFG should then be offered an oral glucose tolerance
test to rule out a diagnosis of diabetes. A result below 11 .1
mmol/1 but above 7.8 mmol/1 indicates that the person doesn't
have diabetes but does have IGT
Next question )
B I A ... ·-
·- 21-: - - ...
0 Q20
Cushing's disease
Conn's syndrome
Adrenal hyperplasia
Submit answer
Reference ranges v
Cushing's disease
Conn's syndrome
Adrena I hyperplasia
With ectopic ACTH from small cell lung cancer, cortisol would not be
suppressed by low dose or high-dose dexamethasone.
r "' I 11
, I tt Discuss (2) I Improve l
Cushing's syndrome: investigations
Investigations are divided into confirming Cushing's synd rome and then
localising the lesion. A hypokalaemic metabolic alkalosis may be seen,
along with impaired glucose tolerance. Ectopic ACTH secretion (e.g.
secondary to small cell lung cancer) is characteristically associated
with very low potassium levels. An insulin stress test is used to
differentrate between true Cushing's and pseudo-Cushing's
Localisation tests
CRH stimulation
• if pituitary source then cortisol rises
• if ectopic/adrenal then no change in cortisol
Next question )
B / A ,.. 1-
~
- - ,..
1..:1 c-:::,
01
Supervised fasting
Submit answer
Reference ranges v
Insulin tolerance test
Supervised fasting
Next question )
CamScanner -! W~ :.i.;,-~ \
lnsulinoma
Basics
• most common pancreatic endocrine tumour
• 10% malignant, 10% multiple
• of patients with multiple tumours, 50% have MEN-1
Features
• of hypoglycaemia: typically early in morning or just before meal,
e.g. diplopia, weakness etc
• rapid weight gain may be seen
• high insulin, raised proinsulin:insulin ratio
• high C-peptide
Diagnosis
• supervised, prolonged fasting (up to 72 hours)
• CT pancreas
Management
• surgery
• diazoxide and somatostatin if patients are not candidates for
surgery
Next question )
I T l ....
0 Q2
Cortisol
Glucagon
Growth hormone
lncretin
Insulin
Submit answer
Reference ranges v
Glucagon
Growth hormone
lncretin
Insulin
Next question )
9' 11
f • Discuss (1) Improve
Next question )
Hypoglycaemia
Causes
• insulinoma - increased ratio of proinsulin to insulin
• self-administration of insulin/sulphonylureas
• liver failure
• Addison's disease
• alcohol
Next question )
B /
Q3
Urinalysis protein+
What average blood glucose level for the past 2 months is this most
likely to represent?
10
11
15
Submit answer
Reference ranges v
A 55-year-old female is reviewed in the diabetes clinic. The following
results are obtained:
Urinalysis protein +
What average blood glucose level for the past 2 months is this most
likely to represent?
10
11
15
Next question )
Q4
Duloxetine
Sodium valproate
Carbamazepine
Tramadol
Submit answer
Reference ranges v
Q4
Duloxetine
Sodium valproate
Carbamazepine
Tramadol
Next question )
Diabetic neuropathy
Diabetes typically leads to sensory loss and not motor loss in peripheral
neuropathy. Painful diabetic neuropathy is a common problem in
clinical practice.
Gastroparesis
• symptoms include erratic blood glucose control, bloating and
vomiting
• management options include metoclopramide, domperidone or
erythromycin (prokinetic agents)
Chronic diarrhoea
• often occurs at night
T4 178 mmol/1
Atrial fibrillation
Lid lag
Pretibial myxoedema
Multinodular goitre
Submit answer
Reference ranges v
Pret161al myxoedema
Multinodular goitre
Next question )
Features
• typical features of thyrotoxicosis
• specific signs limited to Grave's (see below)
Autoantibodies
• TSH receptor stimulating antibodies (90%)
• anti-thyroid peroxidase antibodies (75%)
Q6
Submit answer
Reference ranges v
A 41-year-old woman is investigated for hot flushes and night sweats.
Bloods show a significantly raised FSH level and her symptoms are
attributed to the menopause. Following discussions with the patient
she elects to have hormone replacement treatment. What is the most
significant risk of prescribing an oestrogen-only preparation rather than
a combined oestrogen-progestogen preparation?
Next question )
Hormone replacement therapy: adverse effects
Side-effects
• nausea
• breast tenderness
• fluid retention and weight gain
Potential complications
• increased risk of breast cancer: increased by the addition of a
progestogen
• increased risk of endometrial cancer: reduced by the addition of a
progestogen but not eliminated completely. The BNF states that
the additional risk is eliminated if a progestogen is given
continuously
• increased risk of venous thromboembol ism: increased by the
addition of a progestogen
• increased risk of stroke
• increased risk of ischaemic heart disease if taken more than 1O
years after menopause
Breast cancer
• in the Women's Health Initiative (WHI) study there was a relative
risk of 1.26 at 5 years of developing breast cancer
• the increased risk relates to duration of use
• breast cancer incidence is higher in women using combined
preparations compared to oestrogen-only preparations
• the risk of breast cancer begins to decline when HRT is stopped
and by 5 years it reaches the same level as in women who have
never taken HRT
Next question )
0 Q7
Submit answer
Reference ranges v
Increases peripheral insulin sensitivity
,t •t tt Discuss Improve ]
Next question )
Infertility
• weight reduction if appropriate
• the management of infertility in patients with PCOS should be
supervised by a specialist. There is an ongoing debate as to
whether metformin, clomifene or a combination should be used to
stimulate ovulation
• a 2007 trial published in the New England Journal of Medicine
suggested clomifene was the most effective treatment. There is a
potential risk of multiple pregnancies with anti-oestrogen*
therapies such as clomifene. The RCOG published an opinion
paper in 2008 and concluded that on current evidence metformin
is not a first line treatment of choice in the management of PCOS
• metformin is also used, either combined with clomifene or alone,
particularly in patients who are obese
• gonadotrophins
Tertiary hyperparathyroidism
Secondary hyperparathyroidism
Pseudohypoparathyroidism
Primary hyperparathyroidism
Submit answer
Reference ranges v
Q8
I Tertiary hyperparathyroidism
Secondary hyperparathyroidism
Pseudohypoparathyroidism
Primary hyperparathyroidism
Secondary hyperparathyroidism
• occurs in chronic kidney disease typically
• can be secondary to vitamin D deficiency
• PTH released due to low calcium, high phosphate and lack of
vitamin D activation by diseased kidneys
• PTH level high with calcium levels being low or normal
• medical management primarily: phosphate binders, calcium and
vitamin D supplementation
Tertiary hyperparathyroidism
• autonomous hypersecretion of PTH due to hypertrophied
parathyroid glands
• occurs after a period of long standing secondary
hyperparathyroidism
• treatment involves parathyroidectomy
Next question )
8' f
11
- Discuss (3) Improve
Next question )
Hypercalcaemia: causes
Next question )
Q9
0 5.9
2 8.4
Normal
Diabetes mellitus
Submit answer
Reference ranges v
Impaired glucose tolerance
Next question )
l"n-mabt-1"
Normal 9'Y'CHffllC Control Duioelts--.
H:!Alt Ill ~Z-H l!'mQh nel e ~,I I
0f
c., r.g g\l:oM fl I~ g mmci l
A fasting glucose greater than or equal to 6.1 but less tha n 7.0 mmol/1
implies impaired fasting glucose (IFG)
Diabetes UK suggests:
• 'People with IFG should then be offered an ora l glucose tolerance
test to rule out a diagnosis of diabetes. A result below 11.1
mmol/1 but above 7.8 mmol/1indicates that the person doesn't
Q1
Pituitary tumour
Adrenocortical adenoma
Adrenal carcinoma
Ectopic secretion
Submit answer
Reference ranges v
l Bilateral idiopathic adrenal hyperplasia
Next question )
Primary hyperaldosteronism
Features
• hypertension
• hypokalaemia
o e.g. muscle weakness
o this is a classical feature in exams but studies suggest this
is seen in only 10-40% of patients
• alkalosis
Investigations
• the 2016 Endocrine Society recommend that a plasma
aldosterone/renin ratio is the first-line investigation in suspected
primary hyperaldosteronism
o should show high aldosterone levels alongside low renin
levels (negative feedback due to sodium retention from
aldosterone)
• following this a high-resolution CT abdomen and adrenal vein
sampling is used to differentiate between unilateral and bilateral
sources of aldosterone excess
• Adrenal Venous Sampling (AVS) can be done to identify the gland
secreting excess hormone in primary hyperaldosteronism
Management
• adrenal adenoma: surgery
• bilateral adrenocortical hyperplasia: aldosterone antagonist e.g.
spironolactone
Q2
Hypocal caemia
Rheumatoid arthritis
Coeliac disease
Hypercalcaemia
Submit answer
Reference ranges v
Q2
I Hypocalcaemia
Rheumatoid arthritis
Coeliac disease
Hypercalcaemia
The question gives a slightly atypical history as this is the upper end of
the age range in which patients would be expected to present
1
"' f • Discuss (2) Improve
Next question )
Autoimmune polyendocrinopathy syndrome
Next question )
B / !! A ...
0 Q3
Submit answer
Reference ranges v
I Enlarging pituita ry malign ancy
However, this excludes only primar y adrenal failure and does not
exclude cortiso l deficie ncy secondary to failure of the pituita ry to
produc e ACTH . The correc t answer is therefore pituitary failure due to
damag e by an enlarging malignancy. The other answers all cause
damag e to the adrenal gland.
Next questi on )
Next question )
Next question )
Save my notes
Q4
Submit answer
Reference ranges v
Prednisolone and levothyroxine
Next question )
Next question )
Hypothyroidism: features
General
• Weight gain
• Lethargy
• Cold intolerance
Skin
• Dry (anhydrosis), cold, yellowish skin
• Non-pitting oedema (e.g. hands, face)
• Dry, coarse scalp hair, loss of lateral aspect of eyebrows
Gastrointestinal
• Constipation
Gynaecological
• Menorrhagia
Neurological
• Decreased deep tendon reflexes
• Carpal tunnel syndrome
Next question )
Diabetes mellitus
Normal
Submit answer
Reference ranges v
A 52-year-old man has a set of fasting bloods as part of a work-up for
hypertension. The fasting glucose comes back as 6.5 mmol/1. The test
is repeated and reported as 6. 7 mmol/1. He says he feels constantly
tired but denies any polyuria or polydipsia. How should these results be
interpreted?
Diabetes mellitus
Normal
Next uestion)
Q6
Free T4 29 pmol/1
Anti-nuclear antibodies
Anti-thyroglobulin autoantibodies
Anti-microsomal antibodies
Submit answer
Reference ranges v
TSH < 0.05 mu/ I
Free T 4 29 pmol/ 1
Anti-nuclear antibodies
Anti-thyroglobulin autoantibodies
Anti-microsomal antibodies
Next question )
Next question )
Features
• typical features of thyrotoxicosis
• specific signs limited to Grave's (see below)
Autoantibodies
• TSH receptor stimulating antibodies (90%)
• anti-thyroid peroxidase antibodies (75%)
Next question )
Q7 jc::J
Which one of the following does not have a role in the subsequent
management?
Lugol's iodine
Propranolol
Propylthiouracil
Bicarbonate
Dexamethason e
Submit answer
Free T4 84 pmol/1 (normal range 10-22 pmol/ 1)
Which one of the following does not have a role in the subsequent
management?
Propranolol
Propylthiouracil
Bicarbonate
Dexamethasone
Next question )
Thyroid storm
Precipitating events:
• thyroid or non-thyroidal surgery
• trauma
• infection
• acute iodine load e.g. CT contrast media
Management:
• symptomatic treatment e.g. paracetamol
• treatment of underlying precipitating event
• beta-blockers: typically IV propranolol
• anti-thyroid drugs: e.g. methimazole or propylthiouracil
• Lugol's iodine
• dexamethasone - e.g. 4mg IV qds - blocks the conversion of T4 to
T3
Next question )
'Y
R 1 •
Q8
Free T 4 25 pmol/1
Radioiodine treatment
Carbimazole
Propranolol
Intravenous corticosteroids
Submit answer
Carbimazole
Propranolol
Intravenous corticosteroids
9' f
11 _. Discuss (3) Improve
Next question )
Graves' disease: management
ATD titration
• carbimazole is started at 40mg and reduced gradually to maintain
euthyroidism
• typically continued for 12-18 months
• patients following an ATD titration regime have been shown to
suffer fewer side-effects than those on a block-and-replace
regime
Block-and-replace
• carbimazole is started at 40mg
• thyroxine is added when the patient is euthyroid
• treatment typically lasts for 6-9 months
Radioiodine treatment
• contraindications include pregnancy (should be avoided for 4-6
months following treatment) and age < 16 years. Thyroid eye
disease is a relative contraindication, as it may worsen the
condition
• the proportion of patients who become hypothyroid depends on
the dose given, but as a rule the majority of patient will require
thyroxine supplementation after 5 years
Next question )
Q9
You are conducting the annual review of a 44-year-old woman who has
type 1 diabetes mellitus. You want to assess for diabetic neuropathy
affecting the feet.
Submit answer
Reference ranges v
Test sensation using cotton wool
Next question )
Presentations
• neuropathy: loss of sensation
• ischaemia: absent foot pulses, reduced ankle-brachia! pressure
index (ABPI), intermittent claudication
• complications: calluses, ulceration, Charcot's arthropathy,
cellulitis, osteomyelitis, gangrene
All patients with diabetes should be screened for diabetic foot disease
on at least an annual basis
• screening for ischaemia: done by palpating for both the dorsalis
pedis pulse and posterial tibial artery pulse
• screening for neuropathy: a 1o g monofllament is used on various
parts of the sole of the foot
Moderate
Low risk risk High risk
All patients who are moderate or high risk (I.e. any problems other than
simple calluses) should be followed up regularly by the local diabetic
foot centre.
Next question )
Save my notes J
A 39-year-old woman is reviewed in the clinical pharmacology clinic
following referral by her GP for management of her hypertension. She
has a blood pressure of 159/ 90 mmHg despite 3 oral anti-hypertensive
medications including full dose ramipril. Examination in t he clinic
confirms the elevated blood pressure. Her pulse is 72 and regu lar. Her
chest is clear and her abdomen is soft and non-tender with no palpable
masses. Her body mass index is 28 kg/ m 2 •
Hb 130 g/ 1
K+ 3.1 m mol/I
Bicarbonate 31 mmol/I
Conn's syndrome
Cushing's syndrome
Essential hypertension
Phaeochromocytoma
Cushing's syndrome
Essential hypertension
Phaeochromocytoma
Cushing's syndrome is unlikely given that the body mass index is only
slightly elevated, and the presence of an adrenal adenoma and
biochemical abnormalities effectively rules out essential hypertension.
Phaeochromocytoma may be associated with hypokalaemia, but is
more likely to be associated with episodic hypertension associated with
bursts of catecholamine release. In renal artery stenosis, a significant
rise in creatinine would be expected in association with the introduction
of the ramipril.
Next question )
Primary hyperaldosteronism
Features
• hypertension
• hypokalaemia
o e.g. muscle weakness
o this is a classical feature in exams but studies suggest this
is seen in only 10-40% of patients
• alkalosis
Investigations
• the 2016 Endocrine Society recommend that a plasma
aldosterone/renin ratio is the first-line investigation in suspected
primary hyperaldosteronism
o should show high aldosterone levels alongside low renin
levels (negative feedback due to sodium retention from
aldosterone)
• following this a high-resolution CT abdomen and adrenal vein
sampling is used to differentiate between unilateral and bilateral
sources of aldosterone excess
• Adrenal Venous Sampling (AVS) can be done to identify the gland
secreting excess hormone in primary hyperaldosteronism
Management
• adrenal adenoma: surgery
• bilateral adrenocortical hyperplasia: aldosterone antagonist e.g.
spironolactone
Q11
Submit answer
Reference ranges v
Decrease glucose absorption in the gut
r • I •• I • Discuss I Improve l
[ ,6 •t • Discuss Improve ]
Next question )
SGLT-2 inhibitors
Patients taking SGLT-2 drugs often lose weight, which can be beneficial
in type 2 diabetes mellitus.
Next question )
B I A ... ., _
·- 1-
~
- - ....
Save my notes
Q12
Addison's disease
Submit answer
Reference ranges v
I Latent Autoimmune Diabetes of Adulthood
1
Addison s disease is associated with hypoglycaemia.
8' f
1 - Discuss (5) Improve
Next question )
rised
Diabetes melli tus may be defined as a chronic cond ition char acte
by abnormally raised levels of blood glucose.
?
Why is the man agem ent of diabetes mellitus so important
wou ld
Before the advent of insulin therapy untreated type 1 diab etes
can still
usually result in death. Poorly treated type 1 diab etes mell itus
etic
result in signi fican t morb idity and mort ality (as a resu lt of diab
ent now
ketoacidosis). However, the main focu s of diabetes man agem
se,
is reducing the incidence of mac rova scula r (isch aem ic hear t disea
stroke) and micr ovas cular (eye, nerve and kidney damage)
com plica tions .
Type Notes
Type 1 Auto imm une disor der where the insul in-pr oduc ing
diabetes beta cells of the islets of Langerhans in the
mellitus pancreas are destroyed by the imm une syste m
(T1D M) This results in an abso lute defic iency of insulin
resulting in raised gluco se levels
Patients tend to develop T1 OM in child hood/ early
adult life and typic ally pres ent unwell, poss ibly in
diab etic ketoacidosis
Type 2 This is the mos t com mon cause of diab etes in the
diabetes developed world . It is caused by a relative
mellitus deficiency of insulin due to an excess of adip ose
(T2D M) tissu e. In simp le term s there isn't enough insulin to
'go around' all the excess fatty tissue, leading to
blood gluco se creeping up.
Prediabetes This term is used for patients who don't yet meet
the criteria for a formal diagnosis of T2DM to be
made but are likely to develop the condition over
the next few years . They, therefore, require closer
monitoring and lifestyle interventions such as
weight loss
Type 1 DM Type 2 DM
Remember that the polyuria and polydipsia are due to water being
'dragged' out of the body due to the osmotic effects of excess blood
glucose being excreted in the urine (glycosuria).
Investigations
,,edabetn
Honn.al flVcNmlC control r o-...--.
HoA1c a .&;'-'T nTn:l fflCI ,e c..e ~ l
or
ea1~~ g\lCOM I! 1~ p Ml!'ICl(.I
HbA1c; <a.,r~W)
mmOlmol
Type 1 diabetes
• patients always require insulin to control the blood sugar levels.
This is because there is an absolute deficiency of insulin with no
pancreatic tissue left to stimulate with drugs
• different types of insulin are available according to their duration
of action
Type 2 diabetes
• the majority of patients with type 2 diabetes are controlled using
oral medication
• the first-line drug for the vast majority of patients is metformin
• second-line drugs include sulfonylureas, gliptins and pioglitazone.
Please see the table below for further information
• if oral medication is not controlling the blood glucose to a
sufficient degree then insulin is used
Mechanism of Main side-
Drug class action Route effects Notes
Can be
classified
according to
source
(analogue,
human
sequence and
porcine) and
duration of
action (short,
immediate, long-
acting)
CamScanner ~ Y~ 4!>~1
Metformin Increases insulin Oral Gastrointestinal First-line
sensitivity upset medication in
Decreases Lactic the managemen1
hepatic acidosis* ofT2DM
gluconeogenesis
Cannot be used
in patients with
an eGFR of< 30
ml/min
CamScanner -; Y~ b - ~ I
DPP-4 inhibitors (- Increases Oral Generally well
gliptins) incretin levels tolerated but
which inhibit increased risk
glucagon of pancreatitis
secretion
CamSca nner -; Y~ b - ~ I
Metformin Metformin not tolerated or Cl
Metformin Glitpln
OR
Sulfonylurea
OR
HbA1c > 58 mmovmol (7 5%) Pk>glltazone
!
t.tetformln + gllpt1n HbA1c > 58 mmovmo1 (7.5%}
OR
Metformm + sulfonyrurea
OR
Metform,n + p109htazone Ghplln • p1oghtazone
OR OR
Metformln + SGLT-2 Inhibitor Gllptm + sulfonyturea
OR
PloglttaZone + sulfonylUrea
HbA 1C > 58 mmoVmol (7 5°0)
i
Metformln + sulfonylurea + GLP-1 mimetic C Passmed,cme corn
0 Q13
Diabetes mellitus
lnsulinoma
Nesidioblastosis
Insulin abuse
Sulfonylurea abuse
Submit answer
Reference ranges v
Insulin abuse
Sulfonylurea abuse
Next question )
Hypoglycaemia
Causes
• insulinoma - increased ratio of proinsulln to insulin
• self-administration of insulin/sulphonylureas
• liver failure
• Addison's disease
• alcohol
Next uestion )
Q14
Goitre
Sensorineural deafness
Euthyroid status
Submit answer
Reference ranges v
Short 4th and 5th metacarpals
Sensorineural deafness
Euthyroid status
Next question )
Pendred's syndrome
Ursodeoxycholic acid
Vitamin A
Statins
Fish oil
Fibrates
Submit answer
Reference ranges v
IFibrates
Next question )
Remnant hyperlipidaemia
Overview
• rare cause of mixed hyperlipidaemia (raised cholesterol and
triglyceride levels)
• also known as Fredrickson type Ill hyperlipidaemia, broad-beta
disease and dysbetalipoproteinaemia
• associated with apo-e2 homozygosity
• high incidence of ischaemic heart disease and peripheral vascular
disease
• thought to be caused by impaired removal of intermediate density
lipoprotein from the circulation by the liver
Features
• yellow palmar creases
• palmer xanthomas
• tuberous xanthomas
Management
• flbrates are first line treatment
Next question )
Q16 [Cl
You are on the post-take ward round reviewing a 55-year-old lady who
has been admitted with her third urinary tract infection in as many
months. Of note she is a type II diabetic patient and was commenced
on empagliflozin by her GP four months ago. You suspect recurrent
urinary tract infections secondary to her empagliflozin, a sodium
glucose co-transporter 2 inhibitor used in the treatment of type 2
diabetes mellitus. Where is its main site of action?
Collecting duct
Submit answer
Reference ranges v
Early proximal convoluted tubule
Important side effects to be aware of with this class of drug are genital
tract infections and euglycaemic diabetic ketoacidosis .
Next question )
••
• i passmedicine.com/ quest ion/ qu, * ♦ ➔ 0
SGLT-2 inhibitors
Patients taking SGLT-2 drugs often lose weight, which can be beneficial
in type 2 diabetes mellitus.
Next question )
B / ~
1
2-
= _
-
T
TI T
Save my notes
Search
Q17
pH of 7.38
Ketones 1 + in urine
Glucose of 45 mmol/ I
Submit answer
Reference ranges v
OCI 01 I I OJI I IOIUiilj 01 JI O I I 1031 I 101/ l(Q
Glucose of 45 mmol/1
Clinical features
• General: fatigue, lethargy, nausea and vomiting
• Neurological: altered level of consciousness, headaches,
papilloedema, weakness
• Haematological: hyperviscosity (may result in myocardial
infarctions, stroke and peripheral arterial thrombosis)
• Cardiovascular: dehydration, hypotension, tachycardia
Diagnosis
• 1. Hypovolaemia
• 2. Marked Hyperglycaemia (>30 mmol/L) without significant
ketonaemia or acidosis
• 3. Significantly raised serum osmolarity (> 320 mosmol/kg)
• Note: A precise definition of HHS does not exist, however the
above 3 criteria are helpful in distinguishing between HHS and
OKA. It is also important to remember that a mixed HHS / OKA
picture can occur.
Management
Potassium
• Patients with HHS are potassium deplete but less acidotic than
those with DKA so potassium shifts are less pronounced
• Hyperkalaemia can be present with acute kidney injury
• Patients on diuretics may be profoundly hypokalaemic
• Potassium should be replaced or omitted as required
Next question )
B / /!!j A,..
Save my notes
Q18
Propranolol
Ramipril
Atenolol
Phenoxybenzamine
Doxazosin
Submit answer
Reference ranges v
Ramipril
Atenolol
Phenoxybenzamine
Doxazosin
I
Next question )
'
-•- - - - - •- -- - -- - - - - _._ - -- - -
Phaeochromocytoma
Basics
• bilateral in 10%
• malignant in 10%
• extra-adrenal in 10% (most common site = organ of Zuckerkandl,
adjacent to the bifurcation of the aorta)
Tests
• 24 hr urinary collection of metanephrines (sensitivity 97%*)
• this has replaced a 24 hr urinary collection of catecholamines
(sensitivity 86%)
Next question )
Q19
Metformin
Gliclazide
Acarbose
Sitagliptin
Empagliflozin
Submit answer
Reference ranges v
Acarbose
Sitagliptin
Empagliflozin
Drugs used
• sulphonylureas
• metformin
• alpha-glucosidase inhibitors (acarbose)
• glitazones
• insulin
Next question )
B / tj A ..,. ,-
i:: -- ....
Save my notes
Search
■
Search tex1book ...
Frontal balding
Submit answer
I
Reference ranges v
Multiple bruises on her limbs
Frontal balding
The correct answer is thinning of the axillary hair. The patient has failure
of the adrenal gland due to autoimmune attack. As well as deficiency of
glucocorticoids and mineralocorticoids, she will have lower levels of
androgens, which are usually produced in females by the zona
reticularis of the adrenal cortex. This leads to thinning of hair grown at
puberty, which is androgen dependent. The scalp hair is unaffected.
Stretch marks, skin thinning, easy bruising and poor wound healing are
found in Cushing's disease or in patients given longterm exogenous
steroids. Cafe au lait spots are seen in neurofibromatosis type I.
Next question )
Addison's disease
Features
• lethargy, weakness, anorexia, nausea & vomiting, weight loss, 'salt-
craving'
• hyperpigmentation (especially palmar creases)*, vitiligo, loss of
pubic hair in women, hypotension, hypoglycaemia
• hyponatraemia and hyperkalaemia may be seen
• crisis : collapse, shock, pyrexia
Primary causes
• tuberculosis
• metastases ( e.g. bronchial carcinoma)
• meningococcal septicaemia (Waterhouse-Friderichsen syndrome)
• HIV
• antiphospholipid syndrome
Secondary causes
• pituitary disorders (e.g. tumours, irradiation, infiltration)
Next question )
Q1
Submit answer
Reference ranges v
Increased levels of thyroxine-binding globulin are seen in pregnancy
Next question )
Thyrotoxicosis,
Hypothyroidism
Key points
• thyroxine is safe during pregnancy
• serum thyroid stimulating hormone measured in each trimester
and 6-8 weeks post-partum
• some women require an increased dose of thyroxine during
pregnancy
• breast feeding is safe whilst on thyroxine
Next question )
Carcinoid tumour
Cardiac myxoma
Adrenal carcinoma
Submit answer
Reference ranges v
Small cell lung cancer
Cardiac myxoma
Adrenal carcinoma
Next question )
Next question )
ACTHindependentcauses
• iatrogenic: steroids
• adrenal adenoma (5-10%)
• adrenal carcinoma (rare)
• Carney complex: syndrome including cardiac myxoma
• micronodular adrenal dysplasia (very rare)
Pseudo-Cushing's
• mimics Cushing's
• often due to alcohol excess or severe depression
• causes false positive dexamethasone suppression test or 24 hr
urinary free cortisol
• insulin stress test may be used to differentiate
Next question )
Save mv notes
Q3
Metformln 1g bd
Gliclazide 160mg od
Ploglitazone 45mg od
Aspirin 75mg od
Simvastatin 40mg on
Stop metformin
Stop pioglltazone
Stop gliclazide
Submit answer
Reference ranges v
His medication on admission is as follows:
Metformin 1g bd
Gliclazide 160mg od
Pioglitazone 45mg od
Aspirin 75mg od
Slmvastatin 40mg on
rStop metformin
Stop pioglitazone
Stop gliclazide
Next question )
0 Q4
To!terodine
Teriparatide
Toremifene
Finasteride
Tamsulosin
Submit answer
Reference ranges v
I Tolterodine
Teriparatide
Toremifene
Finasteride
Tamsulosin
•• If • Discuss Improve
Next question )
Urinary incontinence
Risk factors
• advancing age
• previous pregnancy and childbirth
• high body mass index
• hysterectomy
• family history
Classification
• overactive bladder (OAB)/urge incontinence: due to detrusor
overactivity
• stress incontinence: leaking small amount s when coughing or
laughing
• mixed incontinence: both urge and stress
• overflow incontinence: due to bladder outlet obstruction, e.g. due
to prostate enlargement
Initial investigation
• bladder diaries should be completed for a minimum of 3 days
• vaginal examination to exclude pelvic organ prolapse and ability to
initiate voluntary contraction of pelvic floor muscles ('Kegel'
exercises)
• urine dipstick and culture
• urodynamic studies
Episodic hypoglycaemia
Submit answer
Reference ranges v
I Sweat gland hypertrophy
Next question )
Acromegaly: features
Features
• coarse facial appearance, spade-like hands, increase in shoe size
• large tongue, prognathism, interdental spaces
• excessive sweating and oily skin: caused by sweat gland
hypertrophy
• features of pituitary tumour: hypopituitarism, headaches,
bitemporal hemianopia
• raised prolactin in 1/ 3 of cases - galactorrhoea
• 6% of patients have MEN-1
Complications
• hypertension
• diabetes (>10%)
• cardiomyopathy
• colorectal cancer
Q6
A 40-year-old woman complains of feeling tired all the t ime and putting
on weight. On examination a diffuse, non-tender goitre is noted. Blood
tests are ordered:
ESR 14 mm/hr
Pituitary failure
De Quervain's thyroiditis
Hashimoto's thyroiditis
Grave's disease
Submit answer
De Quervain's thyroiditi s
Hashimoto's thyroiditis
Grave's disease
Next question )
Hashimoto's thyroiditis
Features
• features of hypothyroidism
• goitre: firm, non-tender
• anti-thyroid peroxidase and also anti-Tg antibodies
Hypothyroidism Hyperthyrodism
Hashimoto1s
thyroiditis Subacute thyroiditis
{ma) b€- Iii"'} b..c:! II, , .,to•Jt (de Quervain's) Graves' disease
pt,ase II iat,•) (lrutlal 01,,er ll}'pert rolCJ pnase •ouov,eo ~
Lithium
C Passmedlcme corn
Q7
K+ 2.6 mmol/I
Bicarbonate 31 mmol/I
Creatinine 77 µmol/I
Renal ultrasound
Renin:aldosterone ratio
MR angiography
21-hydroxylase estimation
Submit answer
Renin :aldosterone ratio
MR angiography
21-hydroxylase estimation
Next question )
Primary hyperaldosteronism
Features
• hypertension
• hypokalaemia
o e.g. muscle weakness
o this is a classical feature in exams but studies suggest this
is seen in only 10-40% of patients
• alkalosis
Investigations
• the 2016 Endocrine Society recommend that a plasma
aldosterone/renin ratio is the first-line investigation in suspected
primary hyperaldosteronism
o should show high aldosterone levels alongside low renin
levels (negative feedback due to sodium retention from
aldosterone)
• following this a high-resolution CT abdomen and adrenal vein
sampling is used to differentiate between unilateral and bilateral
sources of aldosterone excess
• Adrenal Venous Sampling (AVS) can be done to identify the gland
secreting excess hormone in primary hyperaldosteronism
Management
• adrenal adenoma: surgery
• bilateral adrenocortical hyperplasia: aldosterone antagonist e.g.
spironolactone
Q8
Submit answer
Reference ranges v
l Closes potassium-ATP channels on the beta cell s
When used acutely they increase insulin secretion and decrease insulin
clearance in the liver. However, due to this stimulation of insulin
secretion they can cause hypoglycaemia, the main side effect, and this
can lead to the serious complication of neuroglycopenia.
This resulting lack of glucose supply to the brain can cause confusion
and possible coma. Treatment of this should be through oral glucose,
intramuscular glucagon or intravenous glucose.
Next question )
.. I ,,.,,,,,, I J l '-'~UV'-' I I I lt-'I 'VY'-'
Next question )
Sulfonylureas
Next question )
B I l!!J A ...
Q9
Beta-2 agonist
Beta-1 agonist
Beta-3 agonist
Alpha-1 agonist
Alpha-1 antagonist
Submit answer
Reference ranges v
Beta-1 agonist
Beta-3 agonist
Alpha-1 agonist
Alpha-1 antagonist
Next question )
Ca mSc.an,ner ~ '....,.i;lo-" ~ ~ I
Urinary incontinence
Risk factors
• advancing age
• previous pregnancy and childbirth
• high body mass index
• hysterectomy
• family history
Classification
• overactive bladder (OAB)/ urge incontinence: due to detrusor
overactivity
• stress incontinence: leaking small amounts when coughing or
laughing
• mixed incontinence: both urge and stress
• overflow incontinence: due to bladder outlet obstruction, e.g. due
to prostate enlargement
Initial investigation
• bladder diaries should be completed for a minimum of 3 days
• vaginal examination to exclude pelvic organ prolapse and ability to
initiate voluntary contraction of pelvic floor muscles ('Kegel'
exercises)
• urine dipstick and culture
• urodynamic studies
Management depends on whether urge or stress UI is the predominant
picture. If urge incontinence is predominant:
• bladder retraining (lasts for a minimum of 6 weeks, the idea is to
gradually increase the intervals between voiding)
• bladder stabilising drugs: antimuscarinics are first-line. NICE
recommend oxybutynin (immediate release), tolterodine
(immediate release) or darifenacin (once daily preparation).
Immediate release oxybutynin should, however, be avoided in 'frail
older women'
• mirabegron (a beta-3 agonist) may be useful if there is concern
about anticholinergic side-effects in frail elderly patients
Next question )
B I ~ A ... ·-
1-
i:
- - ...
Save my not es
Search
Q10
Investigations
K+ 5.0 mmol/1
Fluid restriction
IV hydrocortisone
IV normal saline
Oral fludrocortisone
Oral thyroxine
IV hydrocortisone
IV normal saline
Oral fludrocortisone
Oral thyroxine
The hyponatraemia and potassium towards the upper end of the normal
range, coupled with hypoglycaemia, fit well with a diagnosis of
Addison's disease. Although features of hypothyroidism may co-exist
with hypoadrenalism, corticosteroid replacement is the most important
first step in therapy because commencing thyroxine may worsen any
adrenal crisis.
Next question )
Addison's disease
Features
• lethargy, weakness, anorexia, nausea & vomiting, weight loss, 'salt-
craving'
• hyperpigmentation (especially palmar creases)*, vitiligo, loss of
pubic hair in women, hypotension, hypoglycaemia
• hyponatraemia and hyperkalaemia may be seen
• crisis: collapse, shock, pyrexia
Primary causes
• tuberculosis
• metastases (e.g. bronchial carcinoma)
• meningococcal septicaemia (Waterhouse-Friderichsen syndrome)
• HIV
• antiphospholipid syndrome
Secondary causes
• pituitary disorders (e.g. tumours, irradiation, infiltration)
Next question )
Q11
pH 7.25
Bicarbonate 12 mmol/1
pH 7.32
Bicarbonate 17 mmol/1
Stroke
Encephalopathy
Cerebral oedema
Sepsis
Hypokalaemia
I
Submit answer
Reference ranges v
Cerebral oedema
Sepsis
Hypokalaemia
••
11
f • Discuss (4) Improve
Diabetic ketoacidosis
Features
• abdominal pain
• polyuria, polydipsia, dehydration
• Kussmaul respiration (deep hyperventilation)
• Acetone-smelling breath ('pear drops' smell)
Diagnostic criteria
Management
• fluid replacement: most patients with OKA are deplete around 5-8
litres. Isotonic saline is used initially. Please see an example fluid
regime below.
• insulin: an intravenous infusion should be started at 0.1
unit/kg/ hour. Once blood glucose is < 15 mmol/ I an infusion of 5%
unit/ kg/ hour. Once blood glucose is< 15 mmol/ I an infusion of 5%
dextrose should be started
• correction of hypokalaemia
• long-acting insulin should be continued, short-acting insulin
should be stopped
Fluid Volume
Please note that slower infusion may be indicated in young adults (aged
18-25 years) as they are at greater risk of cerebral oedema.
JBDS potassium guidelines
3.5-5.5 40
Next question )
B / ~ A ... ,-
i=: -- ... T I"'
Q12
K+ 4.9 mmol/1
Creatinine 11 O µmol/1
Ceftriaxone + benzylpenicillin
Glucagon
Propranolol
Triiodothyronine
Hydrocortisone
Submit answer
Na+ 131 rnmol/1
K+ 4.9 mmol/1
Glucose 3 .3 mmol/ 1
~ eftriaxone + benzylpenicillin
Glucagon
Propranolol
Triiodothyronine
I Hydrocortisone
[ ,6 If tt Discuss (1 1) Improve ]
Next question )
Q13
Sp ironolactone
Sodium va lproate
Digoxin
Cimetidine
Anabolic steroids
Submit answer
Reference ranges v
Sodium valproate
Digoxin
Cimetidine
Anabolic steroids
Next question )
Gynaecomastia
Causes of gynaecomastia
• physiological: normal in puberty
• syndromes with androgen deficiency: Kallman's, Klinefelter's
• testicular failure: e.g. mumps
• liver disease
• testicular cancer e.g. seminoma secreting hCG
• ectopic tumour secretion
• hyperthyroidism
• haemodialysis
• drugs: see below
Grave's disease
Hashimoto's disease
Iodine deficiency
Solitary adenoma
Submit answer
Solitary adenoma
Next question )
Next question )
Next question )
B I l!j A ,., == 1-
~
=,., T I "'
Save my notes
Search
Q15
Biphasic insulin
Glargine
Detemir
Submit answer
Reference ranges v
lsophane (NPH insulin)
Biphasic insulin
Glargine
Detemir
Endometriosis
Hypothyroidism
Submit answer
Reference ranges v
l Raised uric acid levels
Next question )
Metabolic syndrome
Save my notes
Add exenatide
Add gliclazide
Add pioglitazone
Submit answer
Reference ranges v
Make no changes to management
Add gliclazide
l Add pioglitazone
Pioglitazone is the best option here as it would not put him at risk of
hypoglycaemia, which obviously could be dangerous given his job. The
NICE guidelines would also support the use of a DPP-4 inhibitor (e.g.
sitagliptin or vildagliptin) in this situation.
Next question )
Q18
Corticosteroids
Radioiodine
Block-and-replace regime
Surgery
Submit answer
Reference ranges v
Radioiodine
Block-and-replace regime
surgery
Next question )
B I A ... ·- 1-
~
=. .
Save my notes
Q19
Glycosuria ++
HbA 1 c of 6.7%
Submit answer
Reference ranges v
Fasting glucose 6.8 mmol/L on two occasions
Glycosuria ++
HbA1 c of 6.7%
A 75g oral glucose tolerance test 2 hour value of 8.4 mmol/L would
imply impaired glucose tolerance rather than impaired fasting glucose
Next question )
,.,ediabete:s
NonNI ~ I C c--«il Dsabetas---
f-,q:)Alt 114~-H ll"fflOl'ffl()j 18 c-e .I l
°'
"A •r,,g gu:ow 6 1~ P mmo'l
In 20 I I WHO released supplement ary guidance on the use of AbA Icon
the diagnosis of diabetes:
• a HbA 1 c of greater than or equal to 48 mmol/mol (6.5%) is
diagnostic of diabetes mellitus
• a HbAlc value of less than 48 mmoi/mol (6.5%) does not exclude
diabetes (i.e. it is not as sensitive as fasting samples for detecting
diabetes)
• in patients without symptoms, the test must be repeated to
confirm the diagnosis
• it should be remembered that misleading HbA1 c results can be
caused by increased red cell turnover (see below)
A fasting glucose greater than or equal to 6.1 but less than 7.0 mmoi/1
implies impaired fasting glucose (IFG)
Diabetes UK suggests:
• 'People with IFG should then be offered an ora l glucose tolerance
test to rule out a diagnosis of diabetes. A result below 11.1
mmoi/1 but above 7.8 mmoi/1 indicates that the person doesn't
Q20
Submit answer
Reference ranges v
Between 0.5 to 2.5 mU/ 1
A TSH value between 0.5 to 2.5 mU/ 1is now considered preferable.
Dosage changes should of course also take account of symptoms
Next question )
Hypothyroidism: management
Hypothyroidism: management
Key points
• initial starting dose of levothyroxine should be lower in elderly
patients and those with ischaemic heart disease. The BNF
recommends that for patients with cardiac disease, severe
hypothyroidism or patients over 50 years the initial starting dose
should be 25mcg od with dose slowly titrated. Other patients
should be started on a dose of 50-1 00mcg od
• following a change in thyroxine dose thyroid function tests should
be checked after 8-1 2 weeks
• the therapeutic goal is 'normalisation' of the thyroid stimulating
hormone (TSH) level. As the majority of unaffected people have a
TSH value 0.5-2.5 mU/ I it is now thought preferable to aim for a
TSH in this range
• women with established hypothyroidism who become pregnant
should have their dose increased 'by at least 25-50 micrograms
levothyroxine'* due to the increased demands of pregnancy. The
TSH should be monitored carefully, aiming for a low-normal value
• there is no evidence to support combination therapy with
levothyroxine and liothyronine
Interactions
• iron, calcium carbonate
o absorption of levothyroxine reduced, give at least 4 hours
apart
Q1
A 45-year-old female is reviewed in the medical cli nic with a two month
history of lethargy. Blood tests reveal the following:
K+ 5.1 mmol/1
Creatinine 99 µmol/1
Tot al T4 66 nmol/I
Serum glucose
TSH
Free T4
Submit answer
Reference ranges v
TSH
Free T4
Many labs have an upper reference range for potassium of 5.5 mmol/1,
but in the context of the other results hypoadrenalism shou ld be
suspected
Next question)
Next question )
Next question )
Save my notes
A 54-year-old-woman with known ovarian cancer presents with
confusion. She has become progressively confused over the last few
days, and prior to that, she had started to become constipated. Her
family describes poor oral intake of fluids and poor urinary output as
well. On further discussion with the family, they mention that she was
seen in oncology clinic two weeks ago with results of a bone scan
which they had been told was normal.
Hb 147 g/1
Platelets 321*109/1
WBC 7.8*109/1
K+ 4.7 mmol/ 1
Creatinine 92 µmol/ 1
Osteolytic hypercalcaemia
Calcitriol-mediated hypercalcaemia
Primary hyperparathyroidism
Calcitriol-mediated hypercalcaemia
Primary hyperparathyroidism
Source:
Hypercalcaemia: causes
Next question )
A 66-year-old lady with a long hist ory of poorly control led type-2
diabetes is started on a new medication. She is told it works by
increasing urinary glucose excretion and the doctor says it is an SGLT-2
inhibitor.
Tolbutamide
Dapagliflozin
Exenatide
Linagliptin
Pioglitazone
Submit answer
Reference ranges v
Dapagliflozin
Exenatide
Linagl iptin
Pioglitazone
Tolbutamide is a sulfonylurea.
Pioglitazone is a thiazolidinedione.
Next question)
Q2
Growth hormone
Prolactin
Adrenocorticotrophic hormone
Submit answer
Reference ranges v
Prolactin
Adrenocorticotrophic hormone
Next question )
se being
Prolactin is secreted by the ante rior pitui tary gland with relea
acts as
contr olled by a wide variety of phys iolog ical facto rs. Dopa mine
mine
the prim ary prola ctin releasing inhib itory facto r and henc e dopa
torrh oea.
agon ists such as brom ocrip tine may be used to contr ol galac
to the
It is impo rtant to diffe renti ate the caus es of galac torrh oea (due
astia
actio ns of prola ctin on brea st tissu e) from thos e of gyna ecom
Depression
Polydipsia
Sensory loss
Peptic ulceration
Hypertension
Submit answer
Reference ranges v
Polydipsia
Sensory loss
Peptic ulceration
Hypertension
Next question)
Primary hyperparathyroidism
Q4
132 22
Hb 167 g/1 Na+ Bilirubin pH 7.14
mmol/1 µmol/1
410 * K+ 5.5
Platelets ALP 100 u/1 Pa02 12 KPa
109 / 1 mmol/1
11 .2 * 10.4
WBC Urea ALT 55 u/1 PaC02 1.9 KPa
109 / 1 mmol/1
10.0 * 111
Neuts Creatinine yGT 23 u/1 HC03- 11 µmol/1
109 / 1 µmol/1
1.1 * 3.0
Lymphs Amylase 321 U/ 1 A lbumin 33 g/1 Lactat e
109 / 1 mmol/1
0.1 * c1-
101 Ketones
Eosin Urine Glucose 11 mmol/L
109 / 1 mmol/1 3+
Eosin
109 / 1
er mmol/ 1
Urine
3+
Glucose 11 mmo!/ L
Starvation ketosis
Pancreatitis
Submit answer
Reference ranges v
Metformin induced lactic acidosis
Starvation ketosis
Pancreatitis
8
"' f • Discuss (8) Improve
Next question )
QS
Addison's disease
Prolactinoma
Craniopharyngioma
Submit answer
Reference ranges v
Testicular cancer with brain metastases
Prolactinoma
Craniopharyngioma
Whilst the prolactin level is slightly raised this can be caused by the
pressure effects of the tumour preventing dopamine (which inhibits
prolactin release) from reaching the normal prolactin-producing cells.
Much higher levels would be expected with a prolactinoma.
Next question )
Pituitary adenoma
Investigation requires:
• a pituitary blood profile (including: GH, prolactin, ACTH, FH, LSH
and TFTs)
• formal visual field testing
• MRI brain with contrast
Next question )
B A ... ·-
== 1-
~
=-...,.
Save my notes
Search
Search textbook...
- - -~ · ■
Links
Patient.info
Myeloma
Sarcoidosis
Primary hyperparathyroidism
Vitamin D excess
Osteomalacia
Submit answer
Reference ranges v
Sarcoidosis
Primary hyperparathyroidism
Vitamin D excess
Osteomalacia
Next question)
Q7
Finasteride
Desmopressin
Tamsulosin
Mirabegron
Tolterodine
Submit answer
Reference ranges v
ITolterodine
Next question )
Urinary incontinence
-
Urinary incontinence (UI) is a common problem, affecting around 4-5%
of the population. It is more common in elderly females.
Risk factors
• advancing age
• previous pregnancy and childbirth
• high body mass index
• hysterectomy
• family history
Classification
• overactive bladder (OAB)/urge incontinence: due to detrusor
overactivity
• stress incontinence: leaking small amounts when coughing or
laughing
• mixed incontinence: both urge and stress
• overflow incontinence: due to bladder outlet obstruction, e.g. due
to prostate enlargement
Initial investigation
• bladder diaries should be completed for a minimum of 3 days
• vaginal examination to exclude pelvic organ prolapse and ability to
initiate voluntary contraction of pelvic floor muscles ('Kegel'
exercises)
• urine dipstick and culture
• urodynamic studies
Management depends on whether urge or stress UI is the predominant
picture. If urge incontinence is predominant:
• bladder retraining (lasts for a minimum of 6 weeks, the idea is to
gradually increase the intervals between voiding)
• bladder stabilising drugs: antimuscarinics are first-line. NICE
recommend oxybutynin (immediate release), tolterodine
(immediate release) or darifenacin (once daily preparation).
Immediate release oxybutynin should, however, be avoided in 'frail
older women'
• mirabegron (a beta-3 agonist) may be useful if there is concern
about anticholinergic side-effects in frail elderly patients
Next question)
B I ~ A .... ·-
·- 2= =
1-
- .. T I .... EEEI .... ~ c-::,
-
Save my notes
Search
A 55-year-old man with type 2 diabetes mellitus has his yearly health
check-up at his GP surgery. His HbA1c is 86mmol/L and his GP is
considering adding empagliflozin to help manage his diabetes.
86 Patient's Result
Submit answer
Inhibition of the sodium-glucose transporter in the kidney
Submit answer
Reference ranges v
500 mg at the predawn meal + 1000 mg at the sunset meal
Next question )
Diabetes mellitus: Ramadan
IM Glucagon STAT
Lucozade
100ml IV Dextrose 5%
Submit answer
Reference ranges v
Lucoza de
100ml IV Dextros e 5%
It is not safe to use the oral route when the patient is GCS 5, therefore,
adminis tering Lucozade is not appropriate.
5% dextros e is a mainten ance flu id and not appropr iate for acute
treatme nt of hypogly caemia .
Next questio n )
Next question )
Hypoglycaemia
Causes
• insulinoma - increased ratio of proinsulin to insulin
• self-administration of insulin/sulphonylureas
• liver failure
• Addison's disease
• alcohol
Next question )
1
Q11
Octreotide
External irradiation
Pegvisomant
Trans-sphenoidal surgery
Bromocriptine
Submit answer
Reference ranges v
External irradiation
Pegvisomant
Trans-sphenoidal surgery
Bromocriptine
Next question )
a _ __ _ __ _ _ _ I
•
Next question )
Acromegaly: management
Dopamine agonists
• for example bromocriptine
• the first effective medical treatment for acromegaly, however now
superseded by somatostatin analogues
• effective only in a minority of patients
Somatostatin analogue
• directly inhibits the release of growth hormone
• for example octreotide
• effective in 50-70% of patients
• may be used as an adjunct to surgery
Pegvisomant
• GH receptor antagonist - prevents dimerization of the GH receptor
• once daily s/ c administration
• very effective - decreases IGF-1 levels in 90% of patients to normal
• doesn't reduce tumour volume therefore surgery still needed if
mass effect
Next question )
Q12
Submit answer
Reference ranges v
TSH normal / low; thyroxine low; T3 low
Next question )
Changes are reversible upon recovery from the systemic illness and
hence no treatment is usually needed.
Next question )
Save my notes
Q13
A 31-year-old woman presents for review. For the past few months she
has been feeling generally tired and has not had a normal period for
around 4 months. Prior to this she had a regular 30 day cycle. A
pregnancy test is negative, pelvic examination is normal and routine
bloods are ordered:
FBC Normal
U&E Normal
TFT Normal
ovarian cancer
Turner syndrome
Next question )
Causes
• idiopathic - the most common cause
• chemotherapy
• autoimmune
• radiation
Features are similar to those of the normal climacteric but the actual
presenting problem may differ
• climacteric symptoms: hot flushes, night sweats
• infertility
• secondary amenorrhoea
• raised FSH, LH levels
Next question )
0 Q14
Bromocriptine
Cabergoline
Octreotide
Pegvisomant
Prednisolone
Submit answer
Reference ranges v
Cabergoline
Octreotide
Pegvisomant
Prednisolone
16 8
f • Discuss Improve
Next question )
0 Q15
Submit answer
Reference ranges v
Beta blockers and propylthiouracil
116 11
f • Discuss (5) Improve
Next question )
Thyroid storm
Precipitating events:
• thyroid or non-thyroidal surgery
• trauma
• infection
• acute iodine load e.g. CT contrast media
Management:
• symptomatic treatment e.g. paracetamol
• treatment of underlying precipitating event
• beta-blockers: typically IV propranolol
• anti-thyroid drugs: e.g. methimazole or propylthiouracil
• Lugol's iodine
• dexamethasone - e.g. 4mg IV qds - blocks the conversion of T 4 to
T3
Next question )
Q16
Turner's syndrome
Submit answer
Reference ranges v
Complete androgen insensitivity syndrome
Next question )
Features
• 'primary amennorhoea'
• undescended testes causing groin swellings
• breast development may occur as a result of conversion of
testosterone to oestradiol
Diagnosis
• buccal smear or chromosomal analysis to reveal 46XY genotype
Management
• counselling - raise child as female
• bilateral orchidectomy (increased risk of testicu lar cancer due to
undescended testes)
• oestrogen therapy
Next uestion
Q17
You review a 52-year-old man who is being investigated for weight gain,
impotence and hypertension. On examination you record a blood
pressure of 180/11 O mm Hg and notice purple striae around his
abdomen. He also has some difficulty getting up from a chair and you
observe generalised decreased muscle strength. Routine bloods are
ordered. Given the likely underlying diagnosis, what are the urea and
electrolytes most likely to show?
Submit answer
Reference ranges v
observe generalised decreased muscle strength. Routine bloods are
ordered. Given the likely underlying diagnosis, what are the urea and
electrolytes most likely to show?
Next question )
Q18
Bronchial carcinoid
Hashimoto's thyroiditis
Menopause
Cushing's syndrome
lnsulinoma
Submit answer
Reference ranges v
I lnsulinoma
Next question )
lnsulinoma
Basics
• most common pancreatic endocrine tumour
• 10% malignant, 10% multiple
• of patients with multiple tumours, 50% have MEN-1
Features
• of hypoglycaemia: typically early in morning or just before meal,
e.g. diplopia, weakness etc
• rapid weight gain may be seen
• high insulin, raised proinsulin:insulin ratio
• high C-peptide
Diagnosis
• supervised, prolonged fasting (up to 72 hours)
• CT pancreas
Management
• surgery
• diazoxide and somatostatin if patients are not candidates for
surgery
Q19
Luteal phase
Ovulation
Follicular phase
Menstruation
Submit answer
Reference ranges v
Luteal phase
Ovulation
Follicular phase
Menstruation
Menstrual cycle
Days
Menstruation 1-4
Ovulation 14
-+ @ •©>----.
Atresla
r-,. ~ Single, selected
0
-+ \:2, t ~ - - - - - ~~ tertiary follicle
Selection
0 7 14
iJJ 21 28
Day of merstrual cycle
M en.st:ruatt o n
0 7 14 2i 2B
Day cf menstr ual cy -cle
7 14 21 28
Ovarian - Estrogen
hormone levels - Progesterone
0 7 14 21 28
Day of menstrual cycle
Follicular phase Luteal phase
{proliferative phase) (secretory phase)
Oestradiol levels
also rise again
during the luteal
phase
Next question )
B I l!J
Save my notes
Search
Phaeochromocytoma
lnsulinoma
RET gene
Submit answer
Reference ranges v
lnsulinoma
RET gene
Next question )
Q21
Pituitary MRI
Submit answer
Reference ranges v
Insulin tolerance test
Pituitary MRI
Serum IGF-1 levels are now the first-line test for acromegaly
Importance: 50
Next question )
Acromegaly: investigations
Growth hormone (GH) levels vary during the day and are therefore not
diagnostic.
Serum IGF-1 levels have now overtaken the oral glucose tolerance test
(OGTT) with serial GH measurements as the first-line test. The OGTT
test is recommended to confirm the diagnosis if IGF-1 levels are raised .
Submit answer
Reference ranges v
Growth hormone receptor antagonist
,, f
11 tit Discuss Improve
Next question )
Next question )
Acromegaly: management
Dopamine agonists
• for example bromocriptine
• the first effective medical treatment for acromegaly, however now
superseded by somatostatin analogues
• effective only in a minority of patients
Somatostatin analogue
• directly inhibits the release of growth hormone
• for example octreotide
• effective in 50-70% of patients
• may be used as an adjunct t o surgery
Pegvisomant
• GH receptor antagonist - prevents dimerization of the GH receptor
• once daily s/c administration
• very effective - decreases IGF-1 levels in 90% of patients to normal
• doesn't reduce tumour volume therefore surgery still needed if
mass effect
Metformin
Bromocriptine
Clomifene
Orlistat
Submit answer
Reference ranges v
I Clomifene
Orlistat
Next question )
General
• weight reduction if appropriate
• if a women requires contraception then a combined oral
contraceptive (COC) pill may help regulate her cycle and induce a
monthly bleed (see below)
Hirsutism and acne
• a coc pill may be used help manage hirsutism. Possible options
include a third generation coc which has fewer androgenic
effects or co-cyprindiol which has an anti-androgen action. Both
of these types of COC may carry an increased risk of venous
thromboembolism
• if doesn't respond to COC then topical eflornithine may be tried
• spironolactone, flutamide and fmasteride may be used under
specialist supervision
Infertility
• weight reduction if appropriate
• the management of infertility in patients with PCOS should be
supervised by a specialist. There is an ongoing debate as to
whether metformin, clomifene or a combination should be used to
stimulate ovulation
• a 2007 trial published in the New England Journal of Medicine
suggested clomifene was the most effective treatment. There is a
potential risk of multiple pregnancies with anti-oestrogen*
therapies such as clomifene. The RCOG published an opinion
paper in 2008 and concluded that on current evidence metformin
is not a first line treatment of choice in the management of PCOS
• metformin is also used, either combined with clomifene or alone,
particularly in patients who are obese
• gonadotrophins
Next question )
Q2
Nitrites +++
Blood +
Glucose +++
Ketones
Prot ein
Ketones
Protein
Metformin
Tolbutamide
Dapaglifozin
Sitagliptin
Exanetide
Submit answer
Reference ranges v
I Olbutam1de
Dapaglifozin
Sitagliptin
Exanetide
Next question )
Liraglutide is the other GLP-1 mimetic currently available. One the main
advantages of liraglutide over exenatide is that it only needs to be given
once a day.
I
Consider adding exenatide to metformin and a sulfonylurea if:
• BMI >= 35 kg/ m 2 in people of European descent and there are
problems associated with high weight, or
• BMI < 35 kg/ m 2 and insulin is unacceptable because of
occupational implications or weight loss would benefit other
comorbidities.
Key points
• oral preparation
• trials to date show that the drugs are relatively well tolerated with
no increased incidence of hypoglycaemia
• do not cause weight gain
Next question )
Q3
You are called to see the patient in the acute medical unit as his BM has
dropped to 1.8 mmol/ L.
Starvation
Sulfonylurea-induced hypoglycaemia
Wernicke's encephalopathy
Submit answer
Reference ranges v
Metformin associated lactic acidosis
Starvation
Sulfonylurea-induced hypoglycaemia
Next question)
Next question )
Hypoglycaemia
Causes
• insulinoma - increased ratio of proinsulin to insulin
• self-administration of insulin/sulphonylureas
• liver failure
• Addison's disease
• alcohol
Next question )
B I c-:,
Q4
Submit answer
Reference ranges v
Has failed with insulin therapy
[ ,6 .. .. Discuss Improve ]
Next question )
Liraglutide is the other GLP-1 mimetic currently available. One the main
advantages of liraglutide over exenatide is that it only needs to be given
once a day.
Key points
• oral preparation
• trials to date show that the drugs are relatively well tolerated with
no increased incidence of hypoglycaemia
• do not cause weight gain
Next question )
Save my notes
Search
QS
Anaplastic
Lymphoma
Medullary
Follicular
Papillary
Submit answer
Reference ranges v
Lymphoma
Medullary
Follicular
Papillary
• If tt Discuss Improve
Thyroid cancer
Main points
Type Percentage
Follicular 20%
Further information
Type Notes
Next question )
B I
Save my notes
Search
Q6
Glycogenolysis
Gluconeogenesis
Lipolysis
Submit answer
Reference ranges v
Exchange with hydrogen ions in the colle cting duct s
Gluconeogenesis
Lipolysis
the
The low-insulin cond ition s seen in diab etic keto acid osis stim ulate
-
process of lipolysis and the production of the ketone bodies, beta
bolic
hydroxybutyrate and acetoacetate, which can be used as meta
fuel.
Prolactin level
Karyotype
MRI pituitary
Submit answer
Reference ranges v
Karyotype
MRI pituitary
Klinefelter's? - do a karyotype
Importance: 50
Next question )
Klinefelter's syndrome
Features
• often taller than average
• lack of secondary sexual characteristics
• small, firm testes
• infertile
• gynaecomastia - increased incidence of breast cancer
• elevated gonadotrophin levels but low testosterone
Next question )
Q8
Inferti Iity
Submit answer
Reference ranges v
Increased incidence of breast cancer
Next question )
Klinefelter's syndrome
Features
• often taller than average
• lack of secondary sexual characteristics
• small, firm testes
• infertile
• gynaecomastia - increased incidence of breast cancer
• elevated gonadotrophin levels but low testosterone
Next question )
Q9
Submit answer
Reference ranges v
Blocks thyroid peroxidase from coupling and iodinating the tyrosine
residues on thyrog lobulin
116 f
11 tt Discuss (1) Improve
Next question )
Carbimazole
Mechanism of action
• blocks thyroid peroxidase from coupling and iodinating the
tyrosine residues on thyroglobulin _. reducing thyroid hormone
production
• in contrast propylthiouracil as well as this central mechanism of
action also has a peripheral action by inhibiting 5'-deiodinase
which reduces peripheral conversion of T 4 to T3
Adverse effects
• agranulocytosis
• crosses the placenta, but may be used in low doses during
pregnancy
Q10
17-hydroxylase deficiency
21-hydroxylase deficiency
Submit answer
Reference ranges v
5-alpha reductase deficiency
21 -hydroxylase deficiency
Next question )
CamScanner -! W~ :.i.;,-~\
Congenital adrenal hyperplasia
Overview
• group of autosomal recessive disorders
• affect adrenal steroid biosynthesis
• in response to resultant low cortis ol levels the anter ior pituita ry
secretes high levels of ACTH
• ACTH stimu lates the produ ction of adrenal androgens that may
virilize a female infant
Cause
• 21-hydroxylase deficiency (90%)
• 11 -beta hydroxylase deficiency (5%)
• 17-hydroxylase deficiency (very rare)
- ,I
Proo:, , _
'
..
B
fi' I
!i v-- ''
D
'
,
p
..•
, /
Next question )
Gitelman's syndrome
Features
• normotension
• hypokalaemia
• hypocalciuria
• hypomagnesaemia
• metabolic alkalosis
Next question )
Save my notes
Search
A 29-year-old woman has just found out she is pregnant for the second
time. Her first pregnancy was complicated by gestational diabetes.
Following her first pregnancy she was told she was no longer diabetic.
What is the most appropriate management?
Submit answer
Reference ranges v
Do oral glucose tolerance test as soon as possible after booking
NICE have recently updated their guidelines. Women who are at risk of
gestational diabetes should have an oral glucose tolerance test as soon
as possible after booking, rather than waiting to 16-18 weeks as was
previously advocated.
Time Target
Next question )
B / ~ A . . . ·-
■- 2,-= -- . . .
l
Q13
Glucagonoma
lnsulinoma
Addison's disease
Hypopituitarism
Diabetes mellitus
Submit answer
Reference ranges v
Addison's disease
Diabetes mellit us
Basics
• used in investigation of hypopituitarism
• IV insulin given, GH and cortis ol levels measured
• with norma l pituitary functi on GH and cortis ol should rise
Contraindications
• epilepsy
• ischa emic heart disease
• adrenal insufficiency
You review a 70-year-old who has a long past medical history and is on
multiple drugs. He has developed excessive amounts of breast tissue
bilaterally. Which one of the following drugs is most likely to be
responsible?
Tamoxifen
Terbinafine
Amiodarone
Goserelin (Zoladex)
Lymecycline
Submit answer
Reference ranges v
Score: 0%
Goserelin (Zoladex)
Lymecycline
Gynaecomastia
Causes of gynaecomastia
• physiological : normal in puberty
• syndromes with androgen deficiency: Kallman's, Klinefelter's
• testicular failure: e.g. mumps
• liver disease
• testicular cancer e.g. seminoma secreting hCG
• ectopic tumour secretion
• hyperthyroidism
• haemodialysis
• drugs: see below
Drug causes of gynaecomastia
• spironolactone (most common drug cause)
• cimetidine
• digoxin
• cannabis
• finasteride
• gonadorelin analogues e.g. Goserelin, buserelin
• oestrogens, anabolic steroids
B I A .... ·-
·-
12-: == ... c-:>
=
Save my notes
Search
■
Search textbook...
Q15
Mirabegron
Oxybutynin
Tolterodine
Surgical repair
Submit answer
Reference ranges v
I Mirabegron
Oxybutynin
Tolterodine
Surgical repair
Antich olinerg ics for urge incont inence are associ ated with
confus ion in elderly people - mirabe gron is a prefera ble alterna tive
Importance: 50
The key to this question is to recognise that antimu scarin ics the usual
treatm ent for urge incont inence are contra indicat ed in patien ts with a
history of urinary retention . As such mirabe gron is the correc t answer.
Pelvic floor exercises are used in the treatm ent of stress incont inence
and are unlikely to have an effect in patien ts with pure urge
incontinence.
Surgical repair would be used in patien ts with stress incont inence that
has not improved with pelvic floor exercises and as such is not the
correc t answer.
Glucagon inhibitor
lncretin inhibitor
Alpha-glucosidase inhibitor
Submit answer
L___ _ _ _ _ _ _ _ _ R_e_fe_re_n_c_e_ra_n_g_es_v
_ _ _ _ _ _ _ _ _ _]
Dipeptidyl peptidase-4 (DPP-4) inhibitor
lncretin inhibitor
Alpha-glucosidase inhibitor
Next question )
Q1 7
pH 7.12
HC03 3 mmol/1
Urine dip:
glucose +++
ketones +++
protein
nitrites
Leucocyte esterase -
Submit answer
Reference ranges v
I TSH - normal, T4 - Low, T3 low
Changes are reversible upon recovery from the systemic illness and
hence no treatment is usually needed.
Q18
Alpha-glucosidase inhibitor
Submit answer
Reference ranges v
Alpha-glucosidase inhibitor
Meglitinides
Topical adapalene
Oral clomifene
Topical eflornithine
Topical tazarotene
Submit answer
Reference ranges v
Topical eflornithine
Topical tazarotene
116 11
f • Discuss (1) Improve
Next question )
General
• weight reduction if appropriate
• if a women requires contraception then a combined oral
contraceptive (COC) pill may help regulate her cycle and induce a
monthly bleed (see below)
Infertility
• weight reduction if appropriate
• the management of infertility in patients with PCOS should be
supervised by a specialist. There is an ongoing debate as to
whether metformin, clomifene or a combination should be used to
stimulate ovulation
• a 2007 trial published in the New England Journal of Medicine
suggested clomifene was the most effective treatment. There is a
potential risk of multiple pregnancies with anti-oestrogen*
therapies such as clomifene. The RCOG published an opinion
paper in 2008 and concluded that on current evidence metformin
is not a first line treatment of choice in the management of PCOS
• metformin is also used, either combined with clomifene or alone,
particularly in patients who are obese
• gonadotrophins
Next question )
B / ~
Save my notes
Q20 Jr::"J
Submit answer
Reference ranges v
Increased levels are seen in type 2 diabetes mellitus
[ •• •• • Discuss Improve ]
Sarcoidosis
Primary hyperparathyroidism
Thiazide diuretics
Submit answer
Reference ranges v
Primary hyperparathyroidism
Thiazide diuretics
Next question )
a6 If tt Discuss Improve
Next question )
Hypercalcaemia: causes
Next question )
B I A ...
Q2
T4 188 nmol/1
Hb 14.2 g/ dl
ESR 65 mm/ hr
Hashimoto's thyroiditis
Subacute thyroiditis
Submit answer
Subacute thyroiditis
Next question )
Investigations
• thyroid scintigraphy: globally reduced uptake of iodine-131
Management
• usually self-limiting - most patients do not require treatment
• thyroid pain may respond to aspirin or other NSAI Ds
• in more severe cases steroids are used, particularly if
hypothyroidism develops
Q3
Obesity
Hirsutism
Clitoromegaly
Submit answer
Reference ranges v
Hirsutism
Clitoromegaly
Whilst all of the features listed above may be seen in polycystic ovarian
syndrome, ovarian cysts are the most consistent feature.
Next question )
Next question)
Features
• subfertility and infertility
• menstrual disturbances: oligomenorrhea and amenorrhoea
• hirsutism, acne (due to hyperandrogenism)
• obesity
• acanthosis nigricans (due to insulin resistance)
Investigations
• pelvic ultrasound: multiple cysts on the ovaries
• FSH, LH, prolactln, TSH, and testosterone are useful
investigations: raised LH:FSH ratio is a 'classical' feature but is no
longer thought to be useful in diagnosis. Prolactin may be normal
or mildly elevated. Testosterone may be normal or mildly elevated
- however, if markedly raised consider other causes
• check for impaired glucose tolerance
Next question )
B I fj A .,,. 2=
1- - - ..,.
T I""
0 Q4
A 45-year-old woman with Graves' disease comes for review. She has
recently been diagnosed with thyroid eye disease and is being
considered for radiotherapy. Over the past three days her right eye has
become red and painful. On examination there is proptosis and
erythema of the right eye. Visua l acuity is 6/ 9 in both eyes. What
complication is she most likely to have developed?
Exposure keratopathy
Optic neuropathy
Carbimazole-related neutropaenia
Sjogren's Syndrome
Submit answer
Reference ranges v
exposure Keratopatny
Optic neuropathy
Carbimazole-related neutropaenia
Sjogren's Syndrome
Next question )
Pathophysiology
• it is thought to be caused by an autoimmune response against an
autoantigen, possibly the TSH receptor - retro-orbital
inflammation
• the inflammation results in glycosaminoglycan and collagen
deposition in the muscles
Prevention
• smoking is the most important modifiable risk factor for the
development of thyroid eye disease
• radioiodine treatment may increase the inflammatory symptoms
seen in thyroid eye disease. In a recent study of patients with
Graves' disease around 15% developed, or had worsening of, eye
disease. Prednisolone mav help reduce the risk
Features
• the patient may be eu-, hypo- or hyperthyroid at the time of
presentation
• exophthalmos
• conjunctiva! oedema
• optic disc swelling
• ophthalmoplegia
• inability to close the eyelids may lead to sore, dry eyes. If severe
and untreated patients can be at risk of exposure keratopathy
Management
• topical lubricants may be needed to help prevent corneal
inflammation caused by exposure
• steroids
• radiotherapy
• surgery
Next question )
QS
Regular exercise
Stop smoking
Lose weight
Submit answer
Reference ranges v
A 35-year-old female who has recently being diagnosed with Grave's
disease presents for review 3 months after starting a 'block and replace'
regime with carbimazole and thyroxine. She is concerned about
developing thyiroid eye disease. What is the best way that her risk of
developing thyiroid eye disease can be reduced?
Regular exercise
Stop smoking
Lose weight
Next question )
Q6
Peripheral neuropathy
Cholestasis
Photosensitivity
Weight gain
Submit answer
Reference ranges v
I Weight gain
---------------
16 11
f • Discuss Improve
Next question )
Sulfonylureas
Next question )
Q7
Submit answer
Reference ranges v
A surge of FSH causes ovulation
Next question )
Menstrual cycle
Days
Menstruation 1-4
Ovulation 14
0
© ~
Atresla
{@
le► =~~;
----f-ol-llc.... \.,:.;~~ 'j
COrpLS corpus DegradJn~
alblcans co,pus
0 7 14 21 28
Day of merstrual cycle
M enstruatl c,n
0 7 14 21 28
Day of m,enstruaJ cycl e
1~ 0
7 14 21 28
...g
0
ovarian - Estrogen
:c hormone levels - Progesterone
0 7 14 21 28
Day of menstrual cycle
Follicular phase Luteal phase
(proliferative phase) (secretary phase)
Oestradiol levels
also rise again
during the luteal
phase
Cervical Following menstruation the Under the influence
mucus mucus is thick and forms a of progesterone it
plug across the external os becomes thick,
scant, and tacky
Just prior to ovulation the
mucus becomes clear,
acellular, low viscosity. It
also becomes 'stretchy' - a
quality termed spinnbarkeit
Next question )
B J tj
Save my notes
Search
■
Search textbook...
An 85-year-old comes for review. She has recently had private health
screening and has been advised to see a doctor regarding her thyroid
function tests (TFTs).
Start levothyroxine
Start ca rbimazole
Submit answer
Ca mSc.an,ner ~ '....,.i;lo-" ~ ~ I
Start carbimazole
This patient has subclinical hypothyroidism. By both the TSH and age
criteria advocated by NICE Clinical Knowledge Summaries she should
be monitored for now.
Next question )
Subclinical hypothyroidism
Basics
• TSH raised but T3, T4 normal
• no obvious symptoms
Significance
• risk of progressing to overt hypothyroidism is 2-5% per year
(higher in men)
• risk increased by the presence of thyroid autoantibodies
Management
TSH is between 4 - 10mU/L and the free thyroxine level is within the
normal range
• if< 65 years with symptoms suggestive of hypothyroidism, give a
trial of levothyroxine. If there is no improvement in symptoms,
stop levothyroxi ne
• 'in older people (especially those aged over 80 years) follow a
'watch and wait' strategy, generally avoiding hormonal treatment'
• if asymptomatic people, observe and repeat thyroid function in 6
months
TSH is > 10mU/ L and the free thyroxine level is within the normal range
• start treatment (even if asymptomatic) with levothyroxine if<= 70
years
• 'in older people (especially those aged over 80 years) follow a
'watch and wait' strategy, generally avoiding hormonal treatment'
Next question )
B J f!j A •
~ ! l \ / 0 m\t nntoc- I
Q2
Albumin 41 g/ 1
What is the single most useful test for determining the cause of her
hypercalcaemia?
ESR
Phosphate
Vitamin D level
Parathyroid hormone
ACE level
Submit answer
Reference ranges v
Phosphate
Vitamin D level
Parathyroid hormone
ACE level
Next question )
Q3
K+ 3.0 mmol/1
Creatlnine 65 µmol/1
Bicarbonate 33 mmol/1
Conn's syndrome
Bartter's syndrome
Cushing's syndrome
21-hydroxylase deficiency
Liddle's syndrome
Submit answer
I Bartter's syndrome
Cushing's syndrome
21-hydroxylase deficiency
Liddle's syndro me
Liddle 's syndro me is a rare autosomal domin ant condit ion that causes
hypert ension and hypokalaemic alkalosis. It is though t to be caused by
disordered sodium channels in the distal tubules leading to increased
reabsorption of sodium.
Next question )
Next question )
Bartter's syndrome
Features
• usually presents in childhood, e.g. Failure to thrive
• polyuria, polydipsia
• hypokalaemia
• normotension
• weakness
Next question )
B / /!J T I""
Save mv notes l
Q4 fc:::J
Grave's disease
Hashimoto's thyroiditis
De Quervain's thyroiditis
Submit answer
Papillary cancer of the thyroid
De Quervain's thyroiditis
Grave's disease and toxic multinodular goitre would both present with
hyperthyroidism and papillary thyroid cancer does not produce
thyroxine so would not cause any systemic symptoms.
Investigations
• thyroid scintigraphy: globally reduced uptake of iodine-131
Management
• usually self-limiting - most patients do not require treatment
• thyroid pain may respond to aspirin or other NSAIDs
• in more severe cases steroids are used, particularly if
hypothyroidism develops
Hypothyroidism Hyperthyrodlsm
H shimoco's
th~roic:1•115
Grav s' disease
l1th111m
Q1
Investigations:
Addison disease
Phaeochromocytoma
Primary hyperaldosteronism
Submit answer
Bilateral renal artery stenosis
Phaeochromocytoma
I Primary hyperaldosteronism